CLAT Mock Test 2

You might also like

Download as pdf or txt
Download as pdf or txt
You are on page 1of 70

Sensei Confidential

SENSEI
CLAT MOCK TEST - 2

A Venture by students of NLU Delhi.

1
Sensei Confidential

ENGLISH

Passage (Q.1-Q.5): The announcement of dates for Bengal, Tamil Nadu, Kerala,
Assam and Puducherry assembly elections has focused attention on the strong
repercussions their outcome will have for national politics. BJP governs Assam, is a
strong challenger in Bengal, and its ally AIADMK administers TN. Any setback for
BJP in Bengal and Assam and ally AIADMK in TN could bring the opposition roaring
back into contention in national politics. Consequently, BJP will face greater
questioning over farm policy, high fuel prices and attempts to pursue economic
reforms like privatisation. But a BJP/ NDA victory in these three states will pose
existential questions to its rivals. For instance, a strong opposition voice like Mamata
Banerjee losing sway over Bengal will signal a version of pan-India primacy of BJP.
Not surprisingly, the contestation is fiercest in Bengal, with both sides boasting evenly
matched resources. Foreseeing intense political violence, Election Commission has
announced an eight-phased poll for Bengal, which some fear may only prolong the
unrest. With Mamata invoking Bengali pride to counter the rush of Hindi heartland
leaders campaigning against her, BJP is making a strong pitch for “poriborton” and
attempting to ride anti-incumbency in Bengal.
Much like 2019, PM Modi remains BJP’s most saleable face in Bengal. But an
assembly election may dim Modi’s traction, requiring local leaders to up their game.
With Assam and Bengal elections coinciding,the CAA has forced BJP into a delicate
balancing act. While the party hopes CAA will give it huge tractionamong East Bengal
Hindu migrants, it’s unpopular in Assam where BJP has to underplay it. Delays in
notifying CAA rules can be read as a symptom of BJP’s dilemma, allowing TMC to
dub it a gimmick.
Pursuing a third straight term in TN, AIADMK passed a quota for the influential
Vanniyar community. AIADMK must also grapple with Sasikala’s return from prison,
which could turn into a full blown power struggle if the party loses. TN would worry
BJP for its sizeable industrial base, which will bolster UPA’s fundraising ability if it
wins. Wiped out of Bengal and Tripura, CPM faces a difficult task in Kerala with its
history of voting out incumbents. For Congress, wins in Kerala, Assam, TN and
Puducherry will help regain lost respectability amid constant setbacks and counter
growing dissidence against Rahul Gandhi. The summer of 2021 promises big changes

2
Sensei Confidential

in national politics.

1. What is the passage primarily about?


(a) the fact that Assembly elections in four states could reconfigure national politics
(b) the interrelated political alliances of various parties
(c) the controversies regarding CAA and BJP’s maneuvering it in its favor
(d) the notion of political parties devising policies not for the betterment of their
citizens but to stay inpower

2. These passage mentions the following:


(i) Bengal (ii) Tamil Nadu
(iii) Kerala (iv) Assam
(v) Puducherry
Which of the above are currently governed by the BJP, as per the passage?
(a) All of the above
(b) (i), (ii) and (iv)
(c) (i), (ii), (iii)
(d) only (i)

3. Which of the following is a direct consequence of BJP losing power in any of


the above mentionedStates?
(a) the influence of Mamta will become paramount in Bengal, making her
undefeatable in the subsequent elections
(b) BJP might get a lot of slack for their national policies and governance
(c) BJP might have problems passing policies
(d) might lead to Rahul Gandhi’s political redemption

4. Which of the following is untrue about the impending elections in Bengal?


(a) both the parties are almost at par in the State, and the results can tilt either way
(b) the people of Bengal are likely to vote for BJP because of Modi, but mostly
because of the popularityof the local BJP leaders in Bengal, which is the main
worrying point for Mamta
(c) Both the parties have gotten down to using community based politics

3
Sensei Confidential

(d) all of the above are true

5. Which of the following is


true about the CAA?
(a) CAA was passed by the
BJP government
(b) CAA is widely popular in all the states mentioned above
(c) The delay in notifying the CAA rules might lead to TMC winning the elections
(d) all of the above

Passage (Q.6-Q.9): On Thursday, government introduced sweeping new rules that


encompass a wide spectrum of digital content. But the way in which these rules force-
fit social media, streaming entertainment, and digital news portals all under one
umbrella, is untenable. News content already undertakes compliance with various
standalone legislations. Plus news sites follow print and TV norms, besides the
extensive self-regulation done in multiple layers between journalists and editors every
day. Social media by contrast has run [X], without accountability.
Leave alone discussing these far-reaching changes with stakeholders in digital news,
government has sidestepped even a cursory parliamentary scrutiny by introducing the
sweeping regulatory framework as subordinate legislation. And now, at a time when
news media are already battling adverse economic and legal environments, with
serious charges like sedition being levelled on frivolous grounds, a whole new set of
compliance measures will bring fresh costs and dangers. For example, the oversight
authority set up with government officials and suo motu powers could both encourage
strong-arming by the state and trolls mounting a deluge on a selected media target, to
hurt it punitively.
Government’s aim to impose accountability on social media to tackle a pandemic
of fake news andhatred is sound. But it’s the mainstream news platforms that have
offered the strongest checks on this dangerous phenomenon, by amplifying the
credible facts and information. The three-tier grievanceredressal mechanism proposed
in the rules is only going to put constraints on this work. It is not minimum government,
maximum governance and it’ll not provide benefits to society. It’s best to drop the
current rules. Instead, government can use the ongoing parliamentary exercise in
fleshing out a data protection legislation to meet its objectives of regulating social

4
Sensei Confidential

media content.

6. Which of the following is the underlying theme of the passage?


(a) news platforms must not be clubbed with social media
(b) the new rules must be scraped as they are regressive
(c) the new rules do not serve the purpose the government is trying to solve
(d) the government must unbridle news media outlets

7. How is news content different from the other types of content it is clubbed with under
the new rules?
(a) it goes through various check points, unlike other forms of content
(b) it is more important and relevant than the other forms of content
(c) it is more nation centric than the other forms of content
(d) all of the above

8. What can effectively replace [X] in the above passage?


(a) independent
(b) infamous
(c) rogue
(d) amuck

9. These new rules have comes as a result of which of the following?


(a) the almost negligible accountability of social media sites
(b) to counter the ever-prevalent false information easily available on the internet
(c) the regulate the digital content, which has far reaching effects on the public
(d) to curb sedition against the government

Passage (Q.10-Q.14): Migrants move in search of safer and better lives. They
contribute to the welfare and sustainable development of their countries of origin and
destination. Yet, they face complex human rights challenges and vulnerabilities that
we must address to ensure that no one is left behind.
In the Asia-Pacific region, the number of international migrants has grown from 52
million in 1990 to over 65 million today, roughly 25% of all the world’s international
migrants. Notably, 70% of all international migrants in Asia-Pacific come from within

5
Sensei Confidential

the region.
Most of the region’s migrants send remittances to families and others in their origin
countries. This is important because remittances support household consumption and
contribute to poverty reduction. Between 2009 and 2019, remittances to the region
rose from $183 billion to $330 billion, nearly half ofthe 2019 global total of $717
billion. But since COVID-19, remittances have declined drastically. Those to Eastern
Europe and Central Asia declined over 16% from $57 billion in 2019 to $48 billion in
2020. Remittances in East Asia and the Pacific have fallen more than 10% over the
same period, from $147 billion to $131 billion.
The Asia-Pacific Migration Report 2020 shows that voluntary and involuntary causes
drive migration between countries in Asia and the Pacific and in other regions of the
world. The primary reason is temporary labour migration. Many people also migrate
for education, to escape poverty and inequality, food insecurity and climate change, to
reunite with family or for permanent settlement and retirement. People often move for
more than one reason.
Migrants often lack access to essential services, constrained by laws, fees, language
barriers and restrictions related to residency and migration status. Women migrants,
especially domestic workers, are particularly at risk of discrimination, violence, abuse
and exploitation. Migration-related child protection risks are also a significant concern
throughout Asia and the Pacific.

10. The main theme of the passage revolves around:


(a) In the Asia-Pacific region, the number of international migrants has grown since
1990.
(b) Safe and Orderly Migration in the Asia-Pacific is needed.
(c) Basic statistical scenario of international Migration before and after pandemic.
(d) Rights and exploitation of international migrants throughout Asia and the Pacific.

11. Which of the following option best represent the meaning of the word ‘remittance’ as
has been mentionedin the context of the passage?
(a) a sum of money sent in payment or as a gift.
(b) the use of energy, time, or other resources.
(c) a payment, especially one made by a solicitor to a third party and then claimed
back from the client.

6
Sensei Confidential

(d) the action or process of getting rid of something.

12. Which of the following statements is/are true?


Statement 1: In Eastern Europe and Central Asia, remittances have rose drastically
since COVID-19. Statement 2: Women migrants, especially domestic workers, are
particularly at risk of discrimination, violence, abuse and exploitation.
(a) Statement 1
(b) Statement 2
(c) Statement 1 and 2
(d) Neither Statement 1 nor 2

13. According to the passage, which of the following is the main cause that drive
migration between countriesin Asia and the Pacific and in other regions of the
world?
(a) Education
(b) Food Insecurity and Climate change
(c) Temporary labour migration
(d) To escape poverty and insecurity

14. Which of the following option best represent the meaning of the word ‘involuntary’
as has beenmentioned in the context of the passage?
(a) denoting an outstanding or supreme example of a particular kind of person or
thing.
(b) causing one to lose courage or confidence; disconcerting
(c) done against someone's will; compulsory.
(d) (especially of a position or view) not able to be maintained or defended against
attack or objection.

Passage (Q.15-Q.19): In the early 1900s when legendary musicologist, Pandit V.N.
Bhatkhande, reached the south of India as part of his epic sojourn through the country
to consolidate music systems, he was disappointed.
In accounts of these tours recorded in his travelogue, Majha Dakshinecha Pravaas (My
Musical Journey in Southern India), and in some of his letters and lectures, he

7
Sensei Confidential

expressed his dissatisfaction not only with what he described as “this yaiyy yaiyy style
of singing” but also with the fact that south of India did not quite turn out to be the
land of untouched Shastric music that he was seeking. It was instead a colourful
amalgam that included folk, Islamic and Christian musical styles too. The ardent,
anti-Muslim Sanskritistin him was not pleased.
Bhatkhande was not alone in his conception and more than a hundred years after his
not-so-pleasant discovery, this narrative of the ‘pure South’ continues to drive the
scholarship and public consumption of its classical arts. According to Davesh Soneji,
Associate Professor, Department of South Asia Studies, University of Pennsylvania,
this notion has resulted in the erasure of a rich tradition of Tamil Muslim and Christian
music and many other art forms practised by non-Brahmin or Hindu artist
communities.
At a webinar titled ‘The Arduous Arts: Caste, History, and the Politics of “Classical”
Dance and Music in South India’, organised by the University of California, Los
Angeles, on January 11, Soneji and others examined these and related issues that rarely
get discussed. With conversations about equity gaining prominence, said Professor
Anna Morcom, Mohindar Brar Sambhi Chair of Indian Music, UCLA Department of
Music, “now seemed the right time to run this panel, because of the momentum of
BLM which is causing more discussion and reflection about class amongst South
Asian scholars internationallyand in India… The debates on caste are becoming
more open and heated especially in the world of south Indian classical dance… (and)
it is imperative that these debates are grounded in evidence-based research… I was
really keen to present a panel on caste with a highly authoritative lineup of leading
scholars, performers, and activists,” she said.
The theme of erasure – a natural corollary of hegemony – resonated persistently
throughout its 3.5-hour duration. T M Krishna, singer and activist who has persistently
talked about this issue, raised the questionof ‘aesthetic erasure’, a process by which
[musical] sounds that do not conform to the classical Carnatic style are deemed
plebian and thus undeserving of recognition. It begs the question of what indeed is
“classical” and who decides its parameters.

15. Which of the following statements is/are true with respect to the context of the
passage?
Statement1: Pandit V.N. Bhatkhande, was amused when he reached the south of India

8
Sensei Confidential

as part of his epicsojourn through the country to consolidate music systems.


Statement 2: The accounts of the tours recorded in his travelogue is called, Majha
Dakshinecha Pravaas.
(a) Statement 1
(b) Statement 2
(c) Statement 1 and 2
(d) Neither Statement 1 nor 2

16. Which of the following options best convey the meaning of the word ‘hegemony’ as
has been used in thepassage?
(a) leadership or dominance, especially by one state or social group over others.
(b) the practice or quality of including or involving people from a range of
different social and ethnicbackgrounds and of different genders, sexual
orientations, etc.
(c) (in Hinduism and Buddhism) the sum of a person's actions in this and previous
states of existence, viewed as deciding their fate in future existences.
(d) praise and honour received for an achievement.

17. The main theme of the passage revolves around:


(a) a brief account of a recent webinar organized by the UCLA Center for India and
South Asia, where scholars and artists laid bare the seldom-discussed problem of
caste in the performing arts of South India.
(b) The life and work of legendary musicologist, Pandit V.N. Bhatkhande.
(c) A discussion on ‘aesthetic erasure’, a process by which [musical] sounds that do
not conform to the classical Carnatic style are deemed plebian and thus
undeserving of recognition.
(d) detailed discussion on the erasure of a rich tradition of Tamil Muslim and Christian
music and many other art forms practised by non-Brahmin or Hindu artist
communities.

18. Which of the following best represent the meaning of the word ‘plebian’ as
has been used in thepassage?
(a) member of the general citizenry in ancient Rome as opposed to the privileged
patrician class (mainlyused as a derogatory term for pupils of lower social

9
Sensei Confidential

classes).
(b) the practice of engaging in the same behavior or activity for which one
criticizes another or thepractice of claiming to have moral standards or beliefs
to which one's own behavior does not conform.
(c) a strong supporter of a party, cause, or person.
(d) a lack of cooperation between rival political parties.

19. Given below are the effects of the narrative of the ‘pure South’ which continues to
drive the scholarshipand public consumption of its classical arts. Choose the option
which contains the correct answer.
1) Professor Anna Morcom, Mohindar Brar Sambhi Chair of Indian Music, UCLA
Department of Music,got disappointed when she visited South India and
experienced the cultural environment there.
2) The debates on caste became more open and heated especially in the world of
south Indian classicaldance.
3) South of India did not quite turn out to be the land of untouched Shastric music.
4) Erasure of a rich tradition of Tamil Muslim and Christian music and many other
art forms practised bynon-Brahmin or Hindu artist communities.
Codes:
(a) (2), (3) and (4)
(b) only (1)
(c) (1) and (3)
(d) only (4)

Passage (Q.20-Q.25):

Under the auspices of the Corbett Centre for Maritime Policy Studies comes a
thoughtful book by Jeremy Stocker, Architects of Continental Sea-power: Comparing
Tirpitz and Gorschkov, shedding light on Grand Admiral Tirpitz and Admiral Sergei
Gorschkov, two important naval figures of the 20th century, who wrestled with the
problems and challenges of creating formidable navies for primarily continental
countries.
Tirpitz was a product of Wilhelmine Germany and used his contacts with chancellor
Bulow to satisfy Kaiser Wilhelm’s ambition to make a ‘great power’ of the newly

10
Sensei Confidential

created state. The Kaiser’s passion for heraldry, uniforms, military pomp and
ceremony carried itself forward into a disastrous foreign policy, which would
eventually lead to war and the destruction of the nascent state.
How much of an accomplice in this tragic endeavour was Tirpitz is rightly brought out
by the author, who writes that Tirpitz’s ambition need not necessarily have resulted in
a fleet of battleships, but battleships were large, with an overwhelming presence and
glory. This, unlike, for instance, the U-boats of Germany that eventually did more
damage to the enemy in the first world war.
The author rightly criticises both the Kaiser and Tirpitz for having unsatisfactory
strategic objectives, in building a battle line, other than giving Germany ‘a place in the
sun’ and building a navy of a ‘great power’. Having served in the navy at about the
time that Alfred Thayer Mahan was writing his great work, The Influence of Sea
Power Upon History: 1660–1783, Tirpitz did not seem to have taken Mahan’s caution
of maritime geography, seriously enough.
Although given enough funds to build three capital ships a year, the High Seas Fleet
played little part in influencing the land-centric centre of gravity, permitting as many
as seven infantry divisions to be landed by Britain on the coast of France, without
interference.
Tirpitz’s ambition set off a naval arms race with Britain, in which the latter, with its
vast financialresources, easily outpaced the German building capacity. Eventually,
Britain, the chief trading partner of pre-war Germany, opted for a strategy of a distant
blockade, which crushed the morale of the German people and forced the yet
undefeated German army to sue for peace.
Tirpitz was partly right in foreseeing that building a battle fleet would result in a
Mahanian decisive battle, and so it occurred at Jutland, but it was too little, too late.
The British Grand Fleet though tactically defeated, continued to rule the waves, while
Admiral Hipper was forced to shelter Wilhelms. The author rightly says that if the
funds spent on Tirpitz’s fleet had been given to the army, the German army might have
pulled it off on the Western front.

20. The main theme of the passage revolves around which of the following options?
(a) Discussing and comparing two important naval figures of the 20th century,
who wrestled with theproblems and challenges of creating formidable navies for
primarily continental countries.

11
Sensei Confidential

(b) Discussing modern naval tactics and warfare strategies.


(c) Discussing about how the British Grand Fleet got defeated but continued to
rule the waves, whileAdmiral Hipper was forced to shelter Wilhelms.
(d) Discussion about how the funds spent on Tirpitz’s fleet should have been spent
more effectively andwisely.

21. Which of the following options correctly conveys the meaning of the word
‘nascent’ as has been used inthe passage?
(a) having or showing an excessive interest in or admiration of oneself and one's
physical appearance.
(b) a condition characterized by an extreme tendency to fall asleep whenever in
relaxing surroundings.
(c) (especially of a process or organization) just coming into existence and
beginning to display signs offuture potential.
(d) (of a person or manner) feeling or appearing casually calm and relaxed; not
displaying anxiety,interest, or enthusiasm.

22. Which of the following statement/s is/are true as per the context of the passage?
Statement 1: The Influence of Sea Power Upon History: 1660–1783, was
written by Alfred Thayer Mahan.
Statement 2: The Kaiser’s passion for heraldry, uniforms, military pomp and
ceremony carried itselfforward into a disastrous foreign policy.
(a) Statement 1
(b) Statement 2
(c) Statement 1and 2
(d) Neither Statement 1 nor 2.

23. Which of the following options correctly represent the meaning of the word
‘infantry’ as has been used inthe passage?
(a) soldiers marching or fighting on foot; foot soldiers collectively.
(b) incapable of making mistakes or being wrong.
(c) a general increase in prices and fall in the purchasing value of money.
(d) an arrival or entry of large numbers of people or things.

12
Sensei Confidential

24. Which of the following reasons crushed the morale of the German people and forced
the yet undefeated German army to sue for peace?
(a) The U-boats of Germany eventually did more damage to the enemy in the first world
war.
(b) Tirpitz’s ambition set off a naval arms race with Britain, in which the latter, with
its vast financial resources, easily outpaced the German building capacity and
Britain being the chief trading partner ofpre-war Germany, opted for a strategy of
a distant blockade.
(c) The Kaiser’s passion for heraldry, uniforms, military pomp and ceremony carried
itself forward into a disastrous foreign policy.
(d) As a product of Wilhelmine Germany, Tirpitz used his contacts with chancellor
Bulow to satisfy KaiserWilhelm.

Passage (Q.25-Q.29):

In December 1943, a 20-year-old named Ruth Elias arrived in a cattle car at the
Auschwitz-Birkenau concentration camp. She was assigned to Block 6 in the family
camp, a barracks that housed young women and the camp’s male orchestra, an
ensemble of incarcerated violinists, clarinet players, accordion players and
percussionists who played their instruments not just when the prisoners marched out
for daily labor details, but also during prisoner floggings.
Performances could be impromptu, ordered at the whims of the SS, the paramilitary
guard of the Nazi Party. In a postwar interview, Elias discussed how drunken SS troops
would often burst into the barracks late at night.
Music is often thought of as inherently good, a view exemplified in the playwright
Wilhelm Congreve’s oft-cited aphorism “music hath charms to soothe a savage beast.”
It is also often seen as a form of art that ennobles those who play and listen to it. Its
aesthetic qualities seem to transcend the mundane and horrific.

Yet it’s also been used to facilitate torture and punishment, a topic I think is worth
exploring. When I was researching my book Drunk on Genocide: Alcohol and Mass
Murder in Nazi Germany, I was struck by the ways in which music accompanied deaths
in the camps, the ghettos and the killing fields.
The fusion of alcohol, music and song with mass murder shows how violence was

13
Sensei Confidential

normalised – even celebrated – by the Nazis. Under the Nazi regime, music and song
forged community, camaraderie and shared purpose. In unit bars, around campfires
and at the killing sites, the addition of music was more than just a form of
entertainment. It was also an instrument for promoting a common purpose and
bringing people together. Through rituals of song, drink and dance, the Nazis’ actions
could be collectivised and normalised – and their larger project of violence that much
easier to pull off.
Ultimately, genocide is a societal endeavour; music and song – like political
philosophies – are part of a society’s cultural artefacts. So when mass murder becomes
a central tenet of a society, perhaps it shouldn’t be a surprise that these atrocities are
carried out against a backdrop of stirring song, a rousing military march or a
sentimental Schumann melody.

25. Which of the following option correctly represents the meaning of the word ‘tenet’
as has been mentionedin the passage?
(a) a principle or belief, especially one of the main principles of a religion or
philosophy.
(b) tending to keep a firm hold of something; clinging or adhering closely.
(c) able to be maintained or defended against attack or objection.
(d) very weak or slight.

26. Which of the following statement/s is/are true according to the context of the passage?
Statement 1: The book Drunk on Genocide: Alcohol and Mass Murder in Nazi
Germany, was written byWilhelm Congreve.
Statement 2: Music was an instrument for promoting a common purpose and bringing
people together inthe Nazi regime.
(a) Statement 1
(b) Statement 2
(c) Statement 1 and 2
(d) Neither statement 1 nor 2

27. Which of the following option correctly represents the meaning of the word
‘camaraderie’ as has beenused in the passage?
(a) a small group of people, especially a group of advisers to a ruler or politician, with

14
Sensei Confidential

a shared purpose.
(b) a warning or proviso of specific stipulations, conditions, or limitations.
(c) a dashing and attentive man, especially one acting as a lady's escort.
(d) mutual trust and friendship among people who spend a lot of time together.

28. According to the passage, which one of the following was the main purpose of
music during the Naziregime?
(a) Playing music helped them gain popularity among the masses.
(b) playing music was just a form of entertainment around campfires.
(c) music was used to validate and normalize genocide as a purpose and part of
a society’s culturalartefacts.
(d) for their enjoyment as they held drunken parties.

29. Which of the following writing styles does the author follow in this passage?
(a) Narrative
(b) Persuasive
(c) Expository
(d) Descriptive

Passage(Q.30 -Q.32):

Just before the next round of talks between Indian and Chinese military commanders
to resolve the standoff in eastern Ladakh, US national security adviser Robert O’Brien
has said that time has come to accept that dialogue will not persuade Beijing to change
its ways. O’Brien has a strong point. Beijing’s and extravagant territorial claims have
only been growing in recent years.
That said, counterbalancing an aggressive China through platforms like Quad and
keeping channels of communication open with Beijing should not be seen as mutually
exclusive. Doing so raises the probability of misjudgement and conflict, which is in
nobody’s interest. In this regard, China has in recent
weeks hinted at reviving the 1959 LAC with India. As per this proposal, China will
pretty much retain Aksai Chin in the Ladakh region while giving up its claims on
Arunachal Pradesh.

15
Sensei Confidential

Retaining what each side has and making it the basis for settling the international
border is the best solution to the India-China confrontation. After all, New Delhi has
no hope of wresting Aksai Chin from Beijing just as Chinese claims on Arunachal are
a pipe dream. But the deal needs to be sold to domestic constituencies. Just as it’s said
that only a Republican president such as Richard Nixon could have made a
rapprochement with Mao’s China, Prime Minister Narendra Modi alone has the
capacity to sell an LAC deal in India, as no one will question his nationalist credentials.
With Modi and Xi slated to come face-to- face at the virtual Brics summit next month,
New Delhi should explore if Beijing is serious about the 1959 LAC proposal. If it is,
then Modi could actualise a historic deal. Else, with the situation deadlocked on the
LAC we are heading towards its LoC-isation, which will inflict a heavy military and
economic burden on India.

30. What can most aptly replace [X]?


(a) belligerence (b) standoffishness (c) selfishness (d) peaceful
intrusion

31. Which of the following can be inferred from the passage?


(a) China will be given the possession of the Aksai Chin area after the revival of the
1959 proposal
(b) the revival of the proposal will reinforce the status quo
(c) the revival of the proposal will give unfair gain to China
(d) the revival of the proposal will lead to wrongful gain to India

32. What is the author mean about China’s claim over Arunachal Pradesh being a pipe
dream?
(a) it is a possibility, but not in the near future
(b) it is almost an impossibility
(c) it is a surety, set to happen in near, but uncertain future
(d) it is bound to happen in the coming years

16
Sensei Confidential

GENERAL KNOWLEDGE

Directions for Questions: Each set of questions in this section is based on a single passage.
Please answer each question on the basis of what is stated or implied in the corresponding
passage. In some instances, more than one option may be the answer to the question; in such
a case, please choose the option that most accurately and comprehensively answers the
question.
(33–37) Directions for Questions: Read the passage below and answer the questions that
follow:

Ahead of the launch of India's maiden human spaceflight venture 'Gaganyaan' in December
2021, the Indian Space Research Organisation will undertake two unmanned missions in [1]
and [2], ISRO chairman K Sivan said on Wednesday.
Addressing the inaugural session of a symposium on "Human Spaceflight and Exploration –
Present Challenges and Future Trends," Sivan said the Gaganyaan mission not only aims at
India's maiden human flight to the space but also setting up a new space station. "We are
doing all this (Gaganyaan) on three points – short term plan of two unmanned mission in [1]
and [2], followed by human space flight demonstration in December 2021.
We have the mid term goal of sustaining the human space programme and continuous space
human
presence in space on a new space station", he said.
In this regard, the ISRO has initiated a full–fledged astronaut training facility in close
proximity to Bengaluru for meeting its future requirements. The space agency is also in talks
with Nasa and other space agencies and industries on how it can collaborate on human space
flight and learn from their experience. The Gaganyaan will also assist in Isro's long term goal
of inter–planetary mission.
"Inter–planetary mission is also on the agenda in the long term," the Isro Chief said. On
Gaganyaan mission, Sivan said the space agency has already developed and demonstrated
key technologies such as having an operational launcher with 10–tonne playload capability to
lower orbit, demonstration ofmission design and management, and recovery system like
space qualified parachutes.
"The only missing element were human life science and life support system, which we are
developing now," he said.

17
Sensei Confidential

Isro, Sivan said, has taken onboard many national labs, academic institutions, DRDO labs,
Indian Air Force, CSIR Labas well as many industries as stakeholders for
Gaganyaanprogramme.
Noting that the astronauts have been selected from a pool of Indian Air Force test pilots,
Sivan said the generic space flight training will commence in India shortly.
A mission specific training will be carried out in India using a host of simulators and other
facilities,
he added.

Q.33 Fill [1] and [2]


a) November 2020 and June 2021
b) December 2021 and June 2022
c) December 2020 and July 2021
d) December 2020 and June 2021

Q.34 ISRO has proposed a solar mission to study the Sun’s Corona, photosphere,
chromosphere,
solar emmissions and Coronal Mass Ejections. It will be ISRO’s 2nd space based astronomy
mission
after ASTROSAT which was launched in 2015.Name the solar mission proposed by ISRO.
a) Aditya L2
b) Aditya L1
c) Parker mission
d) None

Q.35 Where will the new HUMAN SPACE FLIGHT CENTRE be built by ISRO
a) Bengaluru, Karnataka
b) Tuticorin, Tamil nadu
c) Chellakere, Tamil Nadu
d) Chellakere, Karnataka

Q.36 Name the communication satellite launched by ISRO for gaganyaan mission
a) IDRS
b) IDRSS

18
Sensei Confidential

c) IRDSS
d) NONE

Q.37 ISRO is planning to build its own space station by which year?
a) 2025
b) 2035
c) 2030
d) 2027

(38–52) Directions for Questions: Read the passage below and answer the questions that
follow:
The phenomenon of defections is not new to Indian politics. It has been plaguing the political
landscape for over five decades. The most prominent case was that of Haryana’s Gaya Lal,
originally an independent MLA who, in 1967, juggled between the Congress and Janata Party
for two weeks, during which he switched his loyalty thrice. The recurrence of this evil
phenomenon led to the 1985 Anti–Defection Law, which defined three grounds of
disqualification of MLAs — giving up party membership; going against party whip; and
abstaining from voting.
Resignation as MLA was not one of the conditions. Exploiting this loophole, the 17 rebel
MLAs in Karnataka resigned, their act aimed at ending the majority of the ruling coalition
and, at the same time, avoiding disqualification. However, the Speaker refused to accept the
resignations and declared them disqualified. This was possible as the legislation empowers
the presiding officer of the House (i.e. the Speaker) to decide on complaints of defection
under no time constraint.
The law originally protected the Speaker’s decision from judicial review. However, this
safeguard was struck down in [1].While the SC upheld the Speaker’s discretionary power, it
underscored that the Speaker functioned as tribunal under the anti–defection law, thereby
making her/his decisions subject to judicial review. This judgment enabled judiciary to
become the watchdog of the anti– defection law, instead of the Speaker, who increasingly had
become a political character contrary to the expected neutral constitutional role. The same
could be witnessed in Shrimanth Balasaheb Patel & Ors vs Speaker Karnataka Legislative
Assembly & Ors (2019), where the three–judge SC bench upheld the then Karnataka
Speaker’s decision of disqualification of the 17 rebel MLAs. However, it struck down his ban
on the MLAs from contesting elections till 2023, negating the only possible permanent

19
Sensei Confidential

solution to the problem. The Supreme Court played the role of a neutral umpire in this
political slugfest. But, the spectacle of MLAs hoarded in a bus, and being sent to a resort,
openly exposed not just the absence of ideological ties between a leader and his party, but
also her/his weak moral character. It was also upsetting to see public acceptance of such
malpractices as part of politics, with some even calling it Chanakya niti!

Q.38) The Anti defection law was introduced by 52nd amendment act,1985 during the tenure
of
Rajiv Gandhi and a new schedule was added to the Indian constitution. What is the schedule
that
incorporates Anti defection law?
a) 10th
b) 9th
c) 8th
d) None

Q.39) By 91st Amendment Act, 2003, the minimum requirement for ‘merger’ was increased
from
one– third members to what?
a) Half of the members
b) At least 90 percent of members
c) Two– third members
d) None of the above.

Q.40) [1] was the case in which Supreme court decided that speaker’s decision can be
reviewed.
Name the case [1].
a) Kihoto Hollohan v. Zachillhu and Others
b) Shrimanth Balasaheb Patel & Ors vs Speaker Karnataka Legislative Assembly & Ors
c) Both
d) None

Q.41) Which committee suggested the exemption from disqualification in case of a split of
political

20
Sensei Confidential

party?
a) Kelkar Committee
b) S.K. Manjhi Committee
c) JVP Committee
d) Dinesh Goswami Committee

Q.42) The constitution can be amended by the Central government except for the basic
features of
the constitution. Which article of the constitution gives this power to the central government?
a) 368
b) 369
c) 371
d) 367

(43–47) Directions for Questions: Read the passage below and answer the questions that
follow:

Parliament on Thursday approved an amendment to the RTI Act that gave the government
powers to decide salary and service terms of the statutory body head and its members, with
Rajya Sabha passing it by a voice vote.
The Upper House witnessed high drama when it negated a motion for sending it to a house
panel.
Speaking on the occasion, Dr Jitendra Singh said that there is no motivation to curtail the
independence of the RTI Act. There is no interference as far as the independence of the Act is
concerned, the Minister assured. Speaking about the fixation of the tenure of Information
Commissioners, he clarified that it is never mentioned in the amendment that the Government
will change it after every two years, as alleged by the Opposition. The Government will not
have unbridled powers to amend the rules, he emphasised. The Minister said that the
Government is open to the suggestions and the amendment is being done with a clean
intention. On the issue of referring it to the Select Committee of the House, the Minister said
that it is the prerogative of the members and it should be determined on the basis of the merit
of the Bill. The existing law says that the public authorities are required to make disclosures
on their organisation, functions and structure powers and duties of its officers and employees
financial information. If such information is not provided by the public authorities on their

21
Sensei Confidential

own, the citizens have the right to demand the same from them under the RTI Act. "Public
authorities" refer to
ministers and government servants among others.
The central information commission is headed by a chief information commissioner and 10
information commissioners. They are appointed by the President (read central government)
appoints them for a fixed tenure of five years and a salary of the rank of the chief election
commissioner and election commissioners respectively.
In the 2019 RTI amendment bill, the government has done away with the fixed tenure of five
years for the chief information commissioners and the information commissioners. Their
salaries too have been altered. Both will now be separately notified by the government of the
day. The amended bill gives the government to fix the salary, which could be lower — given
that the Information Commission is a statutory body unlike the EC which is a constitutional
body. However, the current incumbents’ salaries will not be affected by the amendment.

Q.43) RTI is a legal right for every citizen of INDIA. It was introduced by RTI act of which
year?
a) 2006
b) 2005
c) 2004
d) 2008

Q.44) Who was the first Chief Information Commissioner of India?


a) A.N. Tiwari
b) Deepak Sandhu
c) P.C. Ghosh
d) Wajahat Habibullah

Q.45) Recently Supreme Court held that CJI comes under RTI ACT as he’s a public
authority. It was
decided by a five judge bench. Name the judge who was not a part of this bench.
a) Justice Ranjan Gogoi
b) Justice NV ramana
c) Justice Deepak mishra
d) Justice Sanjeev Khanna

22
Sensei Confidential

Q.46) A high powered committee head by PM Narendra Modi has elected the secretary to the
president as new Central Vigilance Commissioner Name him.
a) Bimal Julka
b) Sanjay Kothari
c) Rajiv Kumar
d) Amitabh bhatt

Q.47) Central vigilance commission was set up in 1964 by the recommendations of which
committee?
a) K Ravikrishnan Committee
b) K Santhanam Committee
c) K Murlitharan Committee
d) K Ramaswami Committee

(Q.48-Q.52):
Keep in mind that the government already has huge powers of surveillance. This was
recognised even in the Justice Srikrishna Committee report that accompanied the draft data
protection law in 2018. So, rather than seeking to revise these powers, the government is
giving itself greater ability to snoop on and interfere with the private lives of citizens. In
particular, the traceability obligation in the new rules is problematic because the technical
literature on this is nearly universal, in agreeing that this would mean breaking the use of
end-to-end encryption for all users on platforms such as WhatsApp.
Also, end-to-end encryption is really needed in the digital economy because data theft and
hacking are only increasing in India. There’s also an issue of platforms themselves misusing
user data. So, ideally, we should be looking to encourage more user-controlled encryption
and not limiting this possibility.

48. Which of the following is not an associated issue with the Rules?
(a) It is of significant concern that the purview of the IT Act, 2000, has been expanded to
bring digital
news media under its regulatory ambit without legislative action.
(b) An intermediary is now supposed to take down content within 36 hours upon receiving
orders from the Government.

23
Sensei Confidential

(c) The rules place fetters upon free speech by fixing the Government as the ultimate
adjudicator of
objectionable speech online.
(d) Regulation has an important place in the scheme of things, and no one advocates giving a
free pass to digital platforms.

49. Which of the following does not depict the positives of the Rules?
(a) Removal of non-consensual intimate pictures within 24 hours,
(b) Publication of compliance reports to increase transparency,
(c) Setting up a dispute resolution mechanism for content removal,
(d) Removing a label to information for users to know whether content is advertised, owned,
sponsored or exclusively controlled to increase public reach.

50. Consider following statements about Three-Tier Redressal Mechanism


I. First handled at the level of the portal itself by its own grievance officer.
II. After satisfactorily settled, it passes on to the self-regulatory body of the sector or industry.
III. If yet not resolved, moves further up to an inter-ministerial oversight committee of the
central government.
Which of these statements is/are correct?
(a) Only I and II
(b) Only II and III
(c) Only I and III
(d) Only III

51. In India, the safe harbour provisions have been defined under which Section of the IT
Act?
(a) Section 43
(b) Section 52
(c) Section 66
(d) Section 79

52. Which of the following statements is not correct about Self-regulation by the self-
regulating bodies of the
publishers?

24
Sensei Confidential

(a) The self-regulating bodies of the publishers should register themselves with the Ministry
of
Information & Broadcasting.
(b) One publisher can have more than one self-regulating bodies.
(c) Such bodies would be headed by a retired judge of the Supreme Court, a High Court, or
an eminent independent person and shall not have more than six members.
(d) It shall also establish an Inter-Departmental Committee for hearing grievances.

(53-57)
A NASA spacecraft, which scientists believe has collected samples from an asteroid, began
its two-year journey back to Earth on Monday. NASA's OSIRIS-REx spacecraft is attempting
to complete a mission to visit [1], a skyscraper-sized asteroid some 320 million km from
Earth, survey the surface, collect samples and deliver them back to Earth. Staff celebrated at
the OSIRIS-Rex control room in Colorado as the space vehicle pushed away from the
asteroid, whose acorn-shaped body formed in the early days of our solar system. OSIRIS-
REx arrived at [1] in [2].
The trip back to Earth will take about two years. The spacecraft will then eject a capsule
containing the asteroid samples, which NASA says will land in a remote area of Utah. NASA
says samples will be distributed to research laboratories worldwide, but 75% of the samples
will be preserved at the Johnson Space Center in Houston for future generations to study with
technologies not yet created.

53. Which of the following has been redacted by [1]?


(a) Pallas
(b) Bennu
(c) Juno
(d) Apophis

54. Which of the following has been redacted by [2]?


(a) 2016
(b) 2017
(c) 2018
(d) 2019

25
Sensei Confidential

55. Which of the following have been discovered by OSIRIS-Rex in [1]?


(a) Hydrogen and Nitrogen molecules
(b) Hydrogen and Oxygen molecules
(c) Oxygen and Nitrogen molecules
(d) Methane and Oxygen molecules

56. In which year was the OSIRIS-Rex misson launched?


(a) 2014
(b) 2015
(c) 2016
(d) 2017

57. What is the name of the robotic-arm of OSIRIS-Rex?


(a) TASCAM
(b) TACSAM
(c) TAGSAM
(d) TASGAM

58 to 63:
The farmers’ protests at the national capital against the three farm laws passed by the
Narendra Modi government have entered the 25th day. At the centre of the protests are two
issues — the fear that minimum support price (MSP) will not be enforced once private
mandis come up; and the amendment of the Agricultural Produce Marketing Committee
(APMC) Act.
Minimum Support Price is a safety net given to the farmers to ensure guaranteed prices and
assured markets. The MSP-based procurement system is aimed to save the crops from price
fluctuations due to various unwarranted factors such as the monsoon, lack of market
integration, information asymmetry and other elements of market imperfection plaguing
Indian agriculture.
However, currently MSP does not have any legal backing. The three new farm laws also do
not have any provision for MSP.
The prices of agricultural commodities often vary due to various factors. If a crop has seen
good harvest season
during a particular year, it may see a sharp fall in its prices. This will lead to farmers

26
Sensei Confidential

withdrawing from sowing the


crop in the next year which will affect the supply. To counter this, MSP is fixed by the
government which is supposed to encourage higher investments and production of crops.
The MSP is fixed twice a year on the recommendations of the [1], which is a statutory body
and submits separate reports recommending prices for kharif and rabi seasons.
Towards this end, the Government announces, Minimum Support Prices (MSP) for 25 major
agricultural commodities each year in both the Crop seasons after taking into account the
recommendations of the [1]. [1]
recommends MSP for twenty two (22) crops and Fair & Remunerative Price (FRP) for [2].
In response to a question in Lok Sabha in March 2020, the Union Minister of Agriculture and
Farmers Welfare [3] had revealed that the [1] considers both ‘A2+FL’ and ‘C2’ costs while
recommending MSP.

Q.58 The Minimum Support Prices (MSP) is fixed twice a year on the recommendations of
the [1], which of the following has been removed with [1] in the passage above?
a Commission for Agricultural Costs and Prices (CACP)
b NITI Aayog
c Cabinet Committee on Economic Affairs
d Ministry of Finance

Q.59 The Minimum Support Prices (MSP) is recommended by the [1] for twenty two (22)
crops and Fair & Remunerative Price (FRP) for [2]. Which of the following crops has been
redacted with [2] in the passage above?
a Wheat
b Sugarcane
c Paddy
d Groundnut

Q.60 Who is the current Union Minister of Agriculture and Farmers Welfare whose name has
been redacted with [3] in the passage above?
a Krishan Pal Gurjaris
b Pralhad Venkatesh Joshi
c Narendra Singh Tomar
d Thaawar Chand Gehlot

27
Sensei Confidential

Q.61 The Union Government has increased the Minimum Support Price (MSP) for Kharif
and Rabi crops of 2019-20 season oilseeds on the line of fixing the MSP at a level of 1.5
times of the cost of production as announced in Union Budget 2018-19. National
Commission on Farmers (NCF) headed by ____________had recommended that the MSP
should be at least 50 percent more than the weighted average cost of production.
a Baldev Singh Dhillon
b Bonbehari Vishnu Nimbkar
c Neelamraju Ganga Prasada Rao
d Dr. M.S. Swaminathan

Q.62 Which of the following is a kharif crop in North India and a rabi crop in South India?
a Wheat
b Jowar
c Sesamum
d Bajra

63-67: As Covid-19 vaccines start being rolled out in several countries, a new strain of the
coronavirus discovered in [1] has sparked panic globally. The strain, which is said to be up to
70 per cent more transmissible than the original, has already forced several countries to shut
their borders or ban travel with the [1] and has sowed fears of further economic disruptions.
The new variant is being called [2] the first "Variant Under Investigation" in the [1] in
December 2020 includes a genetic mutation in the “spike” protein that the SARS-CoV-2
coronavirus uses to infect human cells. According to scientists, these changes, in theory, can
lead to a quicker spread of Covid-19 between people. According to a BBC report, the variant
was first detected in September in the [1]. In November around a quarter of cases in London
were linked to this new variant. The virus-carrying this mutation has so far caused 1,100 new
infections in 60 local authority areas, according to reports quoting [1]Health Secretary Matt
Hancock. [1] stood shut off from the rest of Europe after several nations decided to close
borders over fears of the new coronavirus strain. France became the first to close its border
with the [1] for 48 hours.
India too has joined over two dozen countries in banning inbound travel from the [1]. The
government also said that as a measure of abundant precaution, passengers arriving from the
[1] in all transit flights — those that have taken off or flights which are reaching India before

28
Sensei Confidential

11:59 pm on December 22 — would be subject to mandatory RT-PCR test on arrival at the


respective airports.

Q.63 According to the World Health Organisation, a new strain of the coronavirus discovered
in [1]. Which of the
following countries has been redacted with [1] in the passage above?
a UK
b USA
c China
d Italy

Q.64 What is the name of the new strain of the coronavirus discovered in [1] whose name has
been removed with [2] in the passage above?
a CUI-202012/01
b COVID-202012/01
c COUI-202012/01
d VUI-202012/01

Q.65 The chemical component which is found in most of the virus is


a Proteins
b Carbohydrates
c Vitamins
d Lipids

Q.66 Which of the following statements is not true regarding the Coronaviruses?
a They are a group of DNA viruses that cause diseases in mammals and birds.
b The more lethal diseases are SARS, MERS, and COVID-19
c The genome size of coronaviruses ranges from approximately 26 to 32 kilobases, one of the
largest among RNA viruses.
d They have characteristic club-shaped spikes that project from their surface, which in
electron micrographs create an image reminiscent of the solar corona, from which their name
derives.

Q.67 The study of diseases is called

29
Sensei Confidential

a Paleontology
b Pandmology
c Entomology
d Pathology

68. Who discovered the first ever Virus?


a) Dmitri Ivanovsky
b) Carl Linnaeus
c) Gregor Mendel
d) Anders Celsius

30
Sensei Confidential

LEGAL REASONING
Directions for questions 69-73:
In law, marriage may mean either the acts, agreements, or ceremony by which two persons
enter into wedlock, or their subsequent relation created thereby. In India, marriages are
governed by Personal laws or Family laws. There exist different laws for each religion. Hindu
Marriages are governed according to the Hindu Marriage Act, 1955 and other laws.
According to Hindu Law, marriage is a sacrament or a holy union. Hindu Marriage is not
only a sacrament but also a contract. According to Vedas, a marriage is, the union of flesh
with flesh and bone with bone. Before the enactment of Hindu Marriage Act, 1955, Hindu
marriages were governed by dharmasastra and customs.
A void marriage is no marriage though the formalities" of a marriage might have been gone
through. Some marriages are void from the moment they are entered into, and neither the
wishes of the participants nor their living together can change the situation. There being in
fact no marriage, the court cannot use a discretion to withhold the decree (or rather, the
declaration), even if the petitioner while going through the marriage ceremony knew that
such a marriage was in fact a void one. Under the following circumstances a marriage may be
called void ab initio under
Hindu marriage act 1955-
Living Spouse- When either party has a spouse living at the time of the marriage, such
marriage is void (excepting an Islamic marriage). This implies that the earlier marital tie is
still continuing though the husband and wife may be living separately.
Bigamy -Bigamy is the offence of marrying a second time by one who has a former husband
or wife still living and the marriage between themselves is still continuing. Section 494 of the
Indian Penal Code lays down provisions on punishment for the offence of bigamy that is,
marrying again during life-time of husband or wife. There are two exceptions in cases of-
(a) earlier marriage with such husband or wife having been declared void by a court of
competent jurisdiction; or
(b) such husband or wife at the time of subsequent marriage was absent and unheard of for
seven years continuously and his whereabouts were not known to those who would have
known it had he or she been alive in the meantime. This exception is based on the principles
laid down in section.108 of the Indian Evidence Act, 1872.
Prohibited degrees (Sapinda)- According to Hindu Law, when two persons offer Pinda to the
same ancestor, they are Sapindas to each other. If the parties are within the degrees of

31
Sensei Confidential

prohibited relationship or are sapindas of each other the marriage is void (unless the custom
or usage governing each of them permits of a marriage between them) A custom is a
traditional and widely accepted way of behaving or doing something that is specific to a
particular society, place, or time. Under the aforementioned circumstances, the marriage is
not merely, voidable but void. The 'marriage' simply does not exist. Even it is unnecessary for
the parties to such so-called marriage to obtain a decree of nullity.
A marriage is voidable on either side of the party is known as voidable marriage. It will be
valid unless the petition for invalidating the marriage is made. This marriage is to be declared
void by a competent court under the Hindu Marriage Act, 1955. The parties of such marriage
have to decide whether they want to go with such marriage or make it invalid. The following
are the grounds of voidable marriage under Hindu law
Under-age- If either of the parties has not attained specified age, the marriage between them
shall be void. The specified age for marriage is 18 for female and 21 for male under Hindu
law.
Mental disorder of spouse- The party alleging mental disorder of the spouse must establish
that the mental illness suffered by the spouse is of such kind and extent that it would be
impossible for the petitioner to lead a normal married life and to procreate. {R. Lakshmi
Narayan v. Santhi, (2001)}
Repeated attacks of Insanity- If one party to the marriage undergo repeated attacks of
insanity. Then the marriage can be void at the option of the other party.
Marriage by force or fraud- If one party is forced into a marriage by force or by fraud then
that party can annule the marriage within 1 year from the discovery of force or fraud.
Pregnant with other than the child of the petitioner- If the respondent is pregnant with other
than the child of the petitioner and if the petitioner did not know about this at the time of
marriage, he can annule the marriage.

Q.69 A, a Hindu aged 32 years married Z (Hindu) who resided in England before returning to
India. After 2 years into marriage A realised that Z was already married to X in England and
had not obtained a divorce. A left Z and married M. Z filed a case of bigamy against A.
Decide.
a A is liable for bigamy as he did not divorce Z before marrying M. Thus, he has entered into
bigamous marriage.
b A is liable for bigamy as his marriage with Z was voidable at his option. Therefore, he had
to obtain a decree of nullity.

32
Sensei Confidential

c A will not be liable for bigamy as his previous marriage with Z was void ab initio. Z had a
living spouse at the time of her marriage with A, thus making it a void marriage.
d Instead of A, Z should be charged with bigamy as she was the one who entered into a
marriage while her previous marriage subsisted.

Q.70 A was married to B who was his paternal cousin sister. Both A and B offered pinda to
the same ancestor. Their marriage was held void under prohibited degree of relationship. The
parties to the marriage contended that there have been a couple of instances of cousins getting
married in their family, hence it should be treated as a custom.
Decide
a The marriage will be valid as it is the custom in the family of A and B to marry cousins.
b The marriage will be valid as both A and B are adults and can marry anyone they want to.
c The marriage will be held void as a couple of instances is not sufficient to prove an
existence of custom allowing marriage of Sapindas.
d The marriage will be held voidable at the option of the groom and her family.

Q.71 A took a loan of 1 lakh from B. A was unable to pay back the loan. B coerced A to
marry his daughter to him failing which he would increase the interest by 25%. Marriage took
place 4th April 2020. On 15th May 2020 C daughter of A got to know that her father was
coerced to marry her to B. On 5thApril 2021 she filed a petition to annule the marriage.
Decide.
a It is a void marriage. The marriage stands annulled as it is not valid in the eyes of law.
b It is a voidable marriage, But C cannot annule the marriage as it is more than 1 year from
the date of marriage
c It is a voidable marriage at the option of B as he was compelled to marry her because A
failed to pay the loan.
d It is a voidable marriage at the option of C. The marriage can annulled as it has not been 1
year since she realized about the involvement of force and fraud in her marriage.

Q.72 Pick the option with the correct order of validity of marriages.
(a) A married B. At the time of marriage B was pregnant with the baby of her former
paramour C.
(b) A married B in the year 2000. In the year 2002 he married C without divorcing B.
(c) Z married Y. Y had a disorder which because of which he could not copulate. This came

33
Sensei Confidential

to light only after the marriage.


(d) A married B. One day B left overnight and went missing. A waited for 7 years. A married
C after 8 years after B went missing. B returned soon and filed a case of bigamy against A.
a Voidable, Void, Voidable, Voidable
b Void, Void, Voidable, Voidable
c Voidable, Valid, Void, Voidable
d Voidable, Void, Voidable, Valid

Q.73 Which of the following statements regarding voidable marriage is false?


a Voidable marriage is valid unless the petition for invalidating the marriage is made
b Voidable marriage can be invalidated only at the option of one party.
c The party which does not have the power to invalidate the marriage can simply walk out of
it.
d Marriage of a 20-year old male with an 18 -year-old is an example for voidable marriage.

(74-78) The Supreme Court observed that it is permissible to hold discrete/open enquiry, at
pre FIR Registration stage in corruption cases. In appeal, the bench Justices DY Chandrachud
and MR Shah considered the question whether such an enquiry at pre-FIR stage would be
legal and to whatextent such an enquiry is permissible? Referring to Lalita Kumari vs.
Government of Uttar Pradesh the bench observed thus: an enquiry at pre- FIR stage is held to
be permissible and not only permissible but desirable, more particularly in cases where the
allegations are of misconduct of corrupt practice acquiring the assets/properties
disproportionate to his known sources of income. After the enquiry/enquiry at pre-registration
of FIR stage/preliminary enquiry, if, on the basis of the material collected during such
enquiry, it is found that the complaint is vexatious and/or there is no substance at all in the
complaint, the FIR shall not be lodged. However, if the material discloses prima facie a
commission of the offence alleged, the FIR will be lodged and the criminal proceedings will
be put in motion and the further investigation will be carried out in terms of the Code of
Criminal Procedure. Therefore, such a preliminary enquiry would be permissible only to
ascertain whether cognizable offence is disclosed or not and only thereafter FIR would be
registered.
Therefore, such a preliminary enquiry would be in the interest of the alleged accused also
against whom the complaint is made. The court added that, “It is enough if the information
discloses the commission of a cognizable offence as the information only sets in motion the

34
Sensei Confidential

investigative machinery, with a view to collect all necessary evidence, and thereafter to take
action in accordance with law. Therefore, as such, holding such an enquiry, may be
discrete/open enquiry, at preregistration of FIR stage in the case of allegation of corrupt
practice of accumulating assets disproportionate to his known sources of income, cannot be
said to be per se illegal”. Both Money laundering and Corruption are cognizable offences and
the accused can be arrested by police officers without any warrant.

74. Ramdhari Dinkar, education minister in state of Rajasthan, was indulged in taking an
ample amount of money for transferring teachers and lecturers in government schools. Those
who want to be transferred to their nearby places and schools, has to deposit the mentioned
amount accordingly. For primary school teacher its Rs.15,000, for third grade teacher, its
Rs.20,000; for second grade teacher its Rs.30,000 and for first grade teacher/lecturer its
Rs.50,000. Also if someone wants for deposition can submit Rs.25,000. This all was handled
by his assistant and other ministers at local level. Eventually, the accounting book
maintaining all such records came out in the public domain and the public demands for
enquiry into the case. No complaint as such was filed against the issue but a huge troll starts
over the social media.
Decide-
(a) Preliminary inquiry cannot be taken out by the investigating officers as there is no prima
facie ground available for valid enquiry at pre registration of FIR.
(b) Preliminary inquiry can be taken out by the investigating officers as there is prima facie
ground available for valid enquiry at pre registration of FIR.
(c) Preliminary inquiry cannot be taken out by the investigating officers but based on the
complaint and social media trolls, a case for corruption and money laundering can be made.
(d) None of the above.

75. Dharmendra Textiles and Co. Ltd was dealing in manufacturing and selling of textiles
product and is the biggest leading company in India. The government of India has taken out a
contract with the Company to deliver 1000 tonnes of clothing material rich in Khadi and pure
cotton. The total amounting bill was made of Rs.150 Crore and the government paid Rs.270
Crore and gets the impure material too from the
company to showcase it to be of pure quality and sell it to the customers at government
subsidized retailer shops at fixed price. The opposition party took the matter in discussion in
session of parliament and asked the government about the extra expenditure of Rs.120 crore

35
Sensei Confidential

which was missing from the book of accounts. The government stated the said expenditure to
be incurred in maintaining the situation of COVID-19 under control and is paid from PM
relief fund. A complaint has been made for such enquiry into the case. Decide-
(a) Preliminary enquiry can be taken out by the investigating agency as there is prima facie
evidence
as to the corruption by government in book of accounts.
(b) Preliminary enquiry cannot be taken out by the investigating agency as there is no prima
facie
evidence as to the corruption by government in book of accounts.
(c) Preliminary enquiry can be taken out by the investigating agency as valid complaint has
been filed
against the government and it cannot be done before filing of FIR.
(d) None of the above.

76. Lok Manya Janta Hostipal, a government run hospital in state of Madhya Pradesh, was
one of the best hospitals to cure heart related diseases and had number of experienced
surgeons for the same. One fine day, Mr. Mohan died due to silent heart attack and a
certificate was issued by the local private hospital. His wife took his husband again for check
up in Lok Manya hospital and she was informed that he was alive and she needs to arrange
money for his surgery. She was in shock but had done what she was asked for. She got them
the required medication and prescribed surgery was taken out. She deposited an amount of
Rs.50,000 as surgery fees and later asked for extra amount of Rs.40,000 as her husband is
dead and if she wants his body back. She filed a complaint against the corruption going on in
public hospital and had validly given the death certificate of her husband. Decide-
(a) Preliminary enquiry can be taken based on complaint of women as it is substantial and on
finding of prima facie evidence, subsequent action can be taken against the authorities.
(b) Pre registration of FIR, enquiry cannot be taken in the case as the complaint filed by
women is not substantial and strong prima facie case has to be built for investigation.
(c) Enquiry in the case can be taken only after filing of FIR and there has to be strong
evidence with the lady apart from the death certificate for valid enquiry.
(d) There is no prima facie evidence with the lady to hold out a legal enquiry either
preliminary or pre registering the FIR and hence no action can be there against the
authorities.

36
Sensei Confidential

77. Charul, Priya and Mohini were good friends and were preparing for UPSC exams 2020.
They all cleared the written exams with good pointers and rank and were called for next stage
of selection i.e Interviews. Their interviews were not so satisfactory and all three of them lose
the hopes for getting selected for the post of IAS officers. Soon the selection list was released
and Priya got selected out of the group. Mohini’s father was also a minister and he applied his
approach and asked the selection board to take his child also for the post. He urged them to
adjust some ranks and release another rank list with Mohini’s name.
He also gave a sum of Rs. 3,00,000 for the said purpose. Soon a rectified list was released
with Mohini getting selected for the post. Charul get to know about this adjustment and she
filed a complaint against the same claiming the selection of her friend to be cancelled and
also the board is corrupted in taking huge sums for such posts. Decide-
(a) Preliminary enquiry cannot be taken out in this case and no FIR can be filed for such
corruption
charges as the complaint filed by Charul is vexatious and not valid.
(b) Preliminary enquiry can be taken out in this case and FIR can be filed for such corruption
charges as the complaint filed by Charul is true and valid too.
(c) Preliminary enquiry cannot be taken out in this case but FIR can be filed for such
corruption charges as the complaint filed by Charul is true and valid.
(d) Preliminary enquiry can be taken out in this case but no FIR can be filed for such
corruption charges as the complaint filed by Charul is vexatious and invalid.

78. Choose the most appropriate.


Assertion (A): Enquiry at pre registration of FIR and Preliminary enquiry can be taken out in
cases of
money laundering or corruption under Prevention of Corruption Act.
Reason (R): Both Money laundering and Corruption are cognizable offences and the accused
can be
arrested by police officers without any warrant.
(a) Both A and R is correct and R is the correct explanation of A.
(b) Both A and R is incorrect.
(c) Both A and R is correct but R is not the correct explanation of A.
(d) Only A is correct and R is incorrect.

(79-84)

37
Sensei Confidential

The adoption of the Information Technology (Intermediary Guidelines and Digital Media
Ethics Code) Rules, 2021 (“Intermediary Guidelines”), the government has adopted a
liability centred approach and raised several questions. Content on the internet typically
concerns three parties: (1) ‘content originators’ who post or share content; (2) online
intermediaries who provide a platform for users to share content on; and (3) ‘content
recipients’ who view the content posted by content originators. Conceptually, it is important
to distinguish between websites that post their own content, and online intermediaries that
merely provide a platform .When (inevitably) the material posted by a content originator
violates the law, it raises the question of whether the online intermediary should be held
secondarily liable as well. Section 79(1) of the IT Act states that an online intermediary shall
not be liable for this facilitative role but in case of active promotion of content shall be. This
ensures that intermediaries do not have to scrutinize each user post for potential liability that
may be foisted on them, ensuring the free flow of user content on platforms such as Facebook
and Twitter. However, given the wide reach of the internet, governments also typically want
intermediaries to act as gatekeepers against some of the worst forms of content. As a result, in
India, the immunity offered to intermediaries by Section 79(1) requires intermediaries to
satisfy certain obligations. Crucially, under Section 79(2)(c), online intermediary must
“observe due diligence”. This is where the Intermediary Guidelines become relevant, as
Rules 3 and 4 of the Guidelines prescribe the ‘due diligence’ obligations of online
intermediaries. To do this service providers often take the help on automated regulator and
checkers who review content posted on the platform

79. Sudesh is an Instagram influencer who often posts objectionable content on his profile,
recently he posted a video where he was urging the citizens to protest for the secession of an
Indian state which is an illegal act. This post was reported several times by users but wasn’t
taken down by the app. Instagram claims they have put up a caveat before the video to inform
the viewers about the content. Was the necessary procedure followed?
(a) The process of due diligence was not followed, since the video invited liability it should
have been taken done which was not done.
(b) Caveat or warning put up by Instagram shows that it fulfilled its duty as an intermediary.
(c) No, the steps taken by Instagram were not adequate enough.
(d) The app is a free space and it cannot violate the freedom of expression of the citizens.

80. Aditya is a student of medical sciences and deeply passionate about the legalisation of

38
Sensei Confidential

marijuana for medicinal purposes in India, he often writes articles and makes short videos on
this cause to post on twitter. Dack Jorsey is an internationally acclaimed and very popular
actor and this cause of Aditya resonates with him. He often retweets his tweets and helps him
reach to a wider audience, the right wing government is troubled by this as this is against the
ideology of the ruling party and imposes a fine of twitter for not following due diligence,
were they correct?
(a) Yes, the government was correct as twitter should follow the guidelines laid down by
them
(b) Yes, twitter shall be liable as they failed to follow the prescribed law
(c) No, the content posted on the medium was not objectionable in nature, hence not violative
of the guidelines
(d) None of the above

81. In the wake of a health emergency, twitter became a hub of fake news and rumours which
were also promoted by several celebrities and famous personalities. To ensure that this stops
twitter hires several experts who scrutinise these accounts, their data and personal chats ,
pictures etc, when the news gets out one such personality sues twitter for violation of privacy,
was twitter at fault?
(a) Yes, right to privacy if fundamental and scrutinising information like pictures and chats is
violative of right to privacy
(b) Yes, it failed to maintain due diligence.
(c) No, the app was following the procedure mentioned in the intermediary guidelines, this is
part of
maintaining due diligence.
(d) No, the app was just fulfilling its legal obligation by being a responsible intermediary.

82. Rafael and Roger are very popular celebrities and international stars, it is also very well
known that they are arch rivals. Recently, Roger posted objectionable and vulgar content
about Rafael on his Facebook page, this garnered a lot of attention and millions of likes.
Rafael claims that the content is defamatory and objectionable in nature and along with Roger
also sues Facebook, will Facebook be liable
(a) Yes, Facebook should have removed the content on the request of Rafael
(b) No, the it’s not Facebook’s duty to check whether the nature of the content
(c) Yes, Facebook would be held liable as the process of due diligence was not followed, the

39
Sensei Confidential

content
should have been taken down as it was objectionable in nature
(d) No, only roger would be liable as he posted the defamatory content and Facebook was
just the
intermediary

83. Toland Drump is the chief minister of an Indian state, after losing the 2019 Vidhan Sabha
elections, he was agitated and mobilised his supporters through twitter, his supporters
eventually attacked and pillaged the assembly hall. Will twitter be liable?
(a) Yes, twitter’s actions were not responsible. They should have been more careful.
(b) No, freedom of expression is fundamental in nature. It cannot be abridged.
(c) No, the pillage was not foreseeable by twitter , hence the two events are unrelated.
(d) Yes, the app should have fulfilled its duty by restraining the tweet, due diligence was not
followed and the consequences was a directly related to the tweets.

84. Do these intermediary guidelines violate right to privacy?


(a) Privacy is a fundamental right, by keeping surveillance on the content posted by creators
is violative of right to privacy.
(b) No, to maintain due diligence, it is important to keep a check.
(c) No, Right to privacy would be violated when personal content of individuals is under
check which
includes chats , pictures , contact details etc
(d) None of the above

(85-90)Force majeure has become a fairly common ground to avoid the performance of
contractual obligations in a post-COVD-19 world. At a time when lawyers and parties seek to
examine their force majeure clauses and reassess their contractual arrangements, here is a
report studying how courts have treated force majeure claims. The term ‘force majeure’ has
been defined in Black’s Law Dictionary, as ‘an event or effect that can be neither anticipated
nor controlled. It is a contractual provision allocating the risk of loss if performance becomes
impossible or impracticable, especially as a result of an event that the parties could not have
anticipated or controlled.’
After examining a total of 690 cases, a qualitative set of 177 cases were extracted to form the
core data set for the report. The report categorizes force majeure claims that have been

40
Sensei Confidential

allowed into broad descriptive heads, to give a clearer understanding into the nature of force
majeure claims that have been allowed. These heads are: Orders by the government,
Permission and regulatory clearance, Change in law, Court order, Act of God, Foreign Events
and a residuary category of Disturbances in Contract.
The report finds that of the 177 cases, a force majeure claim was allowed in 68 cases – i.e.
38.4% of the cases we surveyed. Accordingly, a force majeure claim was rejected in the
remaining 109 cases – i.e., 61.5% of the cases we surveyed.
The lockdown measures imposed in India to contain the spread of COVID-19 have had a
naturally disruptive effect on the performance of contracts. With government and regulatory
bodies already invoking force majeure clauses and extending the timelines for performance of
contracts, there is an increasing trend of force majeure claims being made.

85. Aparna books a ticket to New Delhi on an Indigo flight 761. A usual late riser, Aparna
had to stay up all night to board her 5 AM flight for it was very expensive. As Aparna reaches
the airport and checks in her luggage, she is informed that the flight is cancelled due to a
massive locust attack on the Delhi Airport Runway. Indigo neither delayed the flight nor
provided any alternative. Aparna drags Indigo to court and Indigo claims that they did not
breach any contract as it was a situation of force majeure. Decide
(a) Indigo cannot claim force majeure. Even if this was an unexpected event, it does not
fundamentally alter the nature of obligations. They could have delayed the flight.
(b) Indigo can claim force majeure. The unexpected event changes the nature of obligations
of the
contract which was to land in a safe environment.
(c) Indigo cannot claim force majeure. It hurts passenger interest.
(d) Indigo can claim force majeure for Aparna had to wake up early morning.

86. Bombay Dyeing entered into an agreement with Cotton and Co. for purchasing raw
cotton. The contract was through the exchange of letters among them. The seller used to write
them which were confirmed by the buyer. At one instance, Bombay Dyeing wrote 4 letters to
Cotton and Co. during 15th May 2000 to 10 th June 2000. The confirmation for the same was
provided on 20th June 2000. Bombay Dyeing after providing a notice exercised the right to
resale. Bombay Dyeing had also made a claim of the amount of debit note which was Rs.
57,000. The note was returned by Cotton and Co. along with the letter of 20 th June stating
that the contract was void and illegal. Cotton and Co. denied the performance of the

41
Sensei Confidential

contractual obligations. The claim made by Cotton and Co. was the usage of the words usual
force majeure clause in the contract. Decide
(a) The claim is justified due of the existence of a force majeure clause in the contract.
(b) The claim is not justified as the terms involved are vague and uncertain.
(c) The claim is justified as the party did not have control on the unfolding of events.
(d) The claim is not justified on account of non-fulfillment of obligations within a reasonable
timeline.

87. A contract was signed between Vladimir Industries and Gupta Industries and the
governing law of the contract was Indian law. As a result of heightened import duties, the
company’s profit margin was cut down by around 2.5 percent which inconvenienced their
quarterly charts. The CEO of Vladimir wishes to not perform the obligations of the contract
and claim force majeure.
(a) Vladimir can claim force majeure. This is a natural calamity and an unforeseeable event.
(b) Vladimir cannot claim force majeure. This event does not significantly impact their
profitability.
(c) Vladimir can claim force majeure. This event is covered within the dicta of the Supreme
Court.
(d) Vladimir cannot claim force majeure. This event is merely onerous to them and cannot
vitiate
performance of the contract.

88. Krishna Nigam Power Generators is a huge company which generates electricity using
coal and has tie
up with government of Madhya Pradesh to whom it sells the electricity. This company used
to mainly
import coal from Australia as it would cost them cheaper than here in India. Due to change in
government in Australia, the export duty on coal was increased significantly, thus making it
very costly for Krishna Nigam Power Generators to purchase coal from there. Krishna Nigam
Power Generators went to the Supreme Court against the Madhya Pradesh government
claiming relief under force majeure.
Decide whether it will succeed or not?
(a) No, as increase in export duty doesn’t qualify as an event which would make contract
impossible.

42
Sensei Confidential

(b) Yes, it will succeed because their inability to perform the contract arises out of a
government decision and is beyond their control.
(c) No, it will not succeed because it is a not a case of force majeure but a case of frustration
which has made the performance impossible.
(d) Yes, it will succeed because the communication of their inability to perform their part of
the contract due to an unforeseeable event was made as soon as it occurred and not ex post
facto.

89. Mohandas is a merchant based in Chennai deals in cashew, which he procures from
Indian as well as foreign markets mainly from Sri Lanka. Chandanlal, a confectioner in Delhi
enters into a contract with Mohandas for supply of 1000 kilograms of best quality cashew at
the rate of Rs.500 per kilogram every week. The relations between India and Sri Lanka
turned sour and both countries snapped business ties soon after Mohandas agreed to supply
cashew to Chandanlal. Mohandas did not deliver cashew to Chandanlal because he mostly
procured cashew from Sri Lanka, which were comparatively cheaper. He cannot supply to
Chandanlal at the mentioned rates after buying from Indian markets. When asked by
Chandanlal, Mohandas said that due to no trade and war like conditions between Indian and
Sri Lanka he is not in a position to supply him cashew. Chandanlal files a suit for breach of
contract and claims compensation.
(a) Chandanlal will succeed.
(b) Mohandas will succeed.
(c) The contract will be declared void.
(d) The contract shall be re-drafted in existing situations.

90. TSeries contracted with famous singer Mrs. Krishna Murti for Taj Exclusive Club
members' event. After the contract, Mrs. Murti was diagnosed with Covid 19 and was advised
not to perform for the next six months. Despite her willingness, Mrs. Murti could not sing for
Taj Exclusive Club members' event. TSeries filed a suit against Mrs. Krishna Murti for
breach of contract. Decide whether the diagnosis of bronchitis rendered the contract void?
(a) No, as Mrs. Murti was willing to perform for Taj Exclusive Club member's event.
(b) No, as TSeries was not responsible for medical condition of Mrs. Krishna Murti.
(c) Yes, as the medical condition of Mrs. Murti made it a case where the situation is beyond
one’s
control.

43
Sensei Confidential

(d) Yes, as doctor's advice ought to be given preference to contractual obligations.

(91-95) The right to freedom of speech and expression includes right to acquire information
and to disseminate it. Freedom of speech and expression is necessary, for self–expression
which is an important means of free conscience and self–fulfilment. It enables people to
contribute to debates on social and moral issues. It is the best way to find the truest model of
anything, since it is only through it that the widest possible range of ideas can circulate. It is
the only vehicle of political discourse so essential to democracy. Equally important is the role
it plays in facilitating artistic and scholarly endeavors of all sorts. The right to communicate,
therefore, includes right to communicate through any media that is available whether print or
electronic or audio–visual such as advertisement, movie, article, speech etc. That is why
freedom of speech and expression includes freedom of the press. The freedom of the press in
terms includes right to circulate and also to determine the volume of such circulation. This
freedom includes the freedom to communicate or circulate one's opinion without interference
to as large a population in the country, as well as abroad, as is possible to reach. In Indian
Express Newspapers v. Union of India, the role of the media and the essence of its freedoms
was explained by the Supreme Court as follows:
“32..In today's free world, freedom of press is the heart of social and political intercourse.
The press has now assumed the role of the public educator making formal and non–formal
education possible in a large scale particularly in the developing world, where television and
other kinds of modern communication are not still available for all sections of society. The
purpose of the press is to advance the public interest by publishing facts and opinions without
which a democratic electorate cannot make responsible judgments. Newspaper, being
surveyors of news and views having a bearing on public administration, very often carry
material which would not be palatable to governments and other authorities.”
Access to the internet is a basic and essential facet of the freedom of speech and expression
and the Right to Know (including the right of the media to report freely). In addition, access
to the internet is an indispensable requirement for access to various other fundamental rights,
such as access to healthcare and statutory welfare schemes, to which persons are entitled in
law. Today, the internet is an essential and basic attribute of news–reporting. Consequently,
any interference with access to the internet is a direct violation of the right itself. The
Supreme Court has long held that fundamental rights guaranteed under the Constitution also
include ancillary guarantees that make those rights meaningful (PUCL v. Union of India).
Access to the internet was judicially recognized as a fundamental right in a recent decision of

44
Sensei Confidential

the Kerala High Court in Faheema Shirin v. State of Kerala. The National Telcom Policy
2012 also recognizes the right to broadband connectivity as a “basic necessity like education
and health”. In this context, it can be argued that the widespread communication shutdown,
which was enforced by the Government of India in Kashmir from August 4, 2019 onwards,
by removing the internet as a platform, effectively suspended the right itself.

Q.91) What, according to the author, does not fall within the purview of the freedom of
speech and
expression?
a) The right to access the internet.
b) The right to acquire information by all means.
c) The right to disseminate information.
d) The right to freedom of the press.

Q.92) From a reading of the portion of the judgment (in Indian Express Newspapers v. Union
of India) quoted in the above passage, which of the following statements would be the most
amenable to the judges who delivered that judgment?
I. Newspapers, and generally the media, should never be critical of the government in power.
II. The dissemination of facts and opinions is best left to those occupying governmental
positions.
III. The governments of the developing world hardly contribute to formal and informal
education on
a larger scale.
IV. A democratic electorate can make responsible judgments only with the help of the press.
a) Only III
b) Both II and IV
c) Only I
d) None of the above

Q.93) As per the author, which of the following actions by the Government would constitute
a direct
violation of the right to freedom of speech and expression?
I. The Government promulgates a policy whereby access to the internet is curbed between 2
A.M.

45
Sensei Confidential

and 6 A.M. on every Saturday of the month.


II. The Government restricts the access to internet for a specific religious community for
three weeks
following communal riots in the area they reside in.
III. The Government passes a law that gives the right to access internet only to designated
centres in
a city, open to everyone.
a) Only III.
b) Both. II and III.
c) Only I and II.
d) I, II, and III.

Q.94) Fairtel, a national internet service provider, decided to suspend its broadband services
in Pranabespura for a period of three weeks due to reports of cross–border terrorism. The
population of Pranabespura still had access to internet on their phones. Do Fairtel’s actions
constitute a violation of the right to freedom of speech and expression?
a) No, as the population of Pranabespura had access to internet services on its phones.
b) Yes, as no internet service provider has the right to suspend broadband services.
c) Yes, as connectivity to a broadband service is a basic human right.
d) Yes, as restrictions on broadband amount to interference in the access to internet.

Q.95) Article 15(1) of the Constitution of India states that, “The State shall not discriminate
against any citizen on grounds only of religion, race, caste, sex, place of birth or any of
them.” The government of the State of Jaigarh decided to promulgate a law which declared
that there shall be no discrimination on the basis of gender at the time of admission to
primary schools. However, the rate of female foeticide in Jaigarh was reported to be the
highest. In the light of the information provided in the passage, which of the following
statements is true?
I. The State of Jaigarh has ensured that Article 15(1) is ensured to everyone at the time of
admission
to primary schools.
II. The State of Jaigarh should also formulate a policy to deal with the issue of increasing
female
foeticide, as Article 15(1) has not been fully realised.

46
Sensei Confidential

III. The State of Jaigarh has done nothing to ensure compliance with Article 15(1).
a) Only I.
b) Both I and II.
c) Only II.
d) Both I and III.

(96-)Maintaining that female and male officers should be treated equally, the Supreme Court
on March 17, 2020 cleared permanent commission for women in the Navy and asked the
Centre to complete the modalities within three months. A bench headed by Justice D.Y.
Chandrachud said there cannot be 101 excuses for not granting gender equality in the armed
forces and a level playing field is needed.
Denying permanent commission to women officers who have served the nation would result
in a serious miscarriage of justice, it said. The bench, also comprising Justice Ajay Rastogi,
rejected the Centre’s stand that sea sailing duties cannot be granted to SSC (Short Service
Commission) women officers in the Navy because its Russian vessels do not have
washrooms for them. Such arguments, the court said, are contrary to the Centre’s policy of
1991 and 1998 which lifted the statutory bar on the induction of women officers in the Navy.
The bench quashed the prospective effect of the policy barring women officers inducted
before 2008 from being granted permanent commission in the Navy. It also granted pension
benefits to women officers who have retired and were not granted permanent commission. It
said there cannot be gender discrimination in granting permanent commission to women
officers in the Navy after the statutory bar was lifted by the Centre to allow entry of women.
“Once statutory bar was lifted to allow entry of women officers then male and female officers
are to be treated equally in granting permanent commission,” the court said. It said that there
is enough documentary evidence to suggest women officers in the Navy have brought
accolades to the force.
Article 14 of the Constitution states:
“14. The State shall not deny to any person equality before the law or the equal protection of
the laws within the territory of India.”
Article 15(1) of the Constitution states:
“15. (1) The State shall not discriminate against any citizen on grounds only of religion, race,
caste, sex, place of birth or any of them.”
Article 16(1) of the Constitution states:
“16. (1) There shall be equality of opportunity for all citizens in matters relating to

47
Sensei Confidential

employment or appointment to any office under the State.”

Q.96) In light of the information provided in the passage, which Article of the Constitution
did the
denial of permanent commission to women officers in the Navy violate?
a) Article 15(1)
b) Article 16(1)
c) Both (a) and (b)
d) Article 14

Q.97) Consequent to such a ruling, Mubramaniyam Soni, a lawyer and public figure filed a
writ
petition in the Supreme Court before a higher bench stating that the judgment of the Supreme
Court
is erroneous as it affects all the past decisions of the Navy also. In the light of the information
in the
passage, which of the following statements is true?
I. The decision of the Supreme Court affects all the appointments that have been made by the
Navy
since its inception.
II. The decision of the Supreme Court only affects the appointments which will be made in
the
future.
III. The decision of the Supreme Court makes no impact on any appointment related decision
of the
Navy.
a) Only I.
b) Both I and III.
c) Only II.
d) None of the above.

Q.98) In light of the information provided in the passage, which of the following factors
played the
most important role in the judgment of the Supreme Court?

48
Sensei Confidential

I. The requirement of a level playing field in the Navy.


II. Presence of enough documentary evidence to suggest that women officers in the Navy
have
brought accolades to the force.
III. Female and male officers need to be treated equally.
a) Only II
b) Both I and III
c) Both I and II
d) I, II and III

Q.99) Which of the following statements cannot be inferred from the passage?
I. The 1991 and 1998 policy of the Centre did not allow for the permanent commission of
women
officers inducted into the Navy.
II. The only reason women officers weren’t granted permanent commission in the Navy was
because
the Navy’s vessels did not have washrooms for them.
III. Prior to the Centre’s policy in 1991 and 1998, women were not inducted into the Navy.
IV. Consequent to the judgment of the Supreme Court, all women who have retired from the
Navy
shall be granted pension benefits.
a) Only IV
b) Both I and III
c) Only II
d) Both II and IV

Q.100) From your reading of the passage, under which broad right can the rights under
Articles 14, 15 and 16 be categorized?
a) Right to freedom of expression
b) Right to equality
c) Right to employment
d) Right to be promoted

(101-106) News of a member of the Upper House of the Parliament to move a resolution in

49
Sensei Confidential

the House on March 18, 2020 has been doing the rounds. The resolution seeks to remove the
phrase “socialism” from the Preamble of the Constitution, arguing that the word is
“redundant” in the current scenario, and that the word should be dropped to create space for
“economic thinking without a particular thought”.
The member pointed out that the word “socialist” was inserted in the Constitution by the 42 nd
Constitution Amendment Act, 1976, when the country was under Emergency and
fundamental rights were suspended. He also said that the move was made without any
discussion or debate in Parliament. “The word is not required at all because the Constituent
Assembly had discussed it in detail and Dr B R Ambedkar had made it clear and the
Assembly had agreed to settle the issue,” he said. He also pointed out:
“Ambedkar had argued that the Constitution is just a mechanism to regulate the work of the
various arms of the states, and that the matters like how the policy of the state and the
society’s organisation in the social and economic side should be matters which must be
decided by people according to the time and circumstances. He argued and won the argument
that it should be left to people to decide.”
The Preamble of Constitution declares India to be a sovereign, socialist, secular and
democratic republic. The terms “socialist” and “secular” had been inserted in the Preamble
later, to qualify the character of the Indian republic, to assert something that was always
understood to have existed. The procedure for amending a provision in the Constitution is
laid out in Article 368:
368. (1) Notwithstanding anything in this Constitution, Parliament may in exercise of its
constituent power amend by way of addition, variation or repeal any provision of this
Constitution in accordance with the procedure laid down in this article.
(2) An amendment of this Constitution may be initiated only by the introduction of a Bill for
the purpose in either House of Parliament, and when the Bill is passed in each House by a
majority of the total membership of that House and by a majority of not less than two–thirds
of the members of that House present and voting, it shall be presented to the President who
shall give his assent to the Bill and thereupon the Constitution shall stand amended in
accordance with the terms of the Bill:
Provided that if such amendment seeks to make any change in—
.........
(e) the provisions of this article, the amendment shall also require to be ratified by the
Legislatures of not less than one–half of the States by resolutions to that effect passed by
those Legislatures before the Bill making provision for such amendment is presented to the

50
Sensei Confidential

President for assent. (only relevant portion extracted)


In the case of State of U.P. v. Dina Nath Shukla, the Supreme Court had held that:
The Preamble of the Constitution which decries source of power from "We the people of
India", i.e. Bharat, envisions an egalitarian social order to integrate all the people with
equality of status, dignity of person and fraternity as a united Bharat and providing them
socio–economic justice, equality of opportunity and status and dignity of person. It is well
settled legal position that Preamble is part of the Constitution and is the basic structure of the
Constitution.
Moreover, in the case of Kesavananda Bharati v. State of Kerala, the Supreme Court held that
the Parliament has power to amend any provision of the constitution, but doing so, the ‘basic
structure’ of the constitution is to be maintained.

Q.101) From a reading of the above passage, what does the move by the member of the
Upper House
of the Parliament seek to do?
I. Remove the word ‘socialist’ from the Preamble.
II. Amend a provision of the Constitution.
III. Amend the basic structure of the Constitution.
IV. Nullify some effect of the 42nd Amendment to the Constitution.
a) Only III
b) Both I and II
c) Both III and IV
d) None of the above

Q.102) Consider that the Bill to remove the word ‘socialist’ from the Preamble is moved in
the Upper
House of the Parliament. Which of the following statements describes the correct procedure
to do
so?
I. The Bill needs to be passed by the Upper House by a simple majority consequent to which,
it needs
to be signed by the President to come into effect.
II. The Bill needs to be passed by both the Houses of the Parliament by a majority and by not
less

51
Sensei Confidential

than two–thirds of the members present and voting, upon which it comes into effect.
III. The Bill needs to be passed by both the Houses by an absolute majority and be signed by
the
President to come into effect.
a) Only II.
b) Only III.
c) Both I and II.
d) None of the above.

Q.103) From a reading of the comments by the member of the Parliament quoted in the third
paragraph of the passage, which of the following inferences can you draw?
a) The member believes that matters of state policy should be decided by the people.
b) The Constitution has no role in determining what the policy of the State should be.
c) The role of the Constitution is to oversee the proper functioning of a democracy such as
India.
d) Dr. B.R. Ambedkar was the only authority in matters of how the Constitution is to be
interpreted.

Q.104) From a reading of the passage, which of the following statements is a safe
conclusion?
I. Prior to the 42nd Amendment, 1976, India was an un–secular and non–socialist country.
II. The Preamble is merely an introduction to and not a part of the Constitution.
III. By the means of Article 368, the Parliament can amend any provision of the Indian
Constitution.
a) Both II and III
b) Only III
c) Only I
d) Both I and II

Q.105) Consider that in addition to removing the word ‘socialist’ from the Preamble, the
member of
the Parliament also seeks to amend certain provisions of Article 368. What is the procedure to
do
so?

52
Sensei Confidential

a) Seek an approval of the President of India


b) Move a resolution in the Lower House of the Parliament
c) The procedure prescribed in Article 368 itself
d) None of the above

53
Sensei Confidential

LOGICAL REASONING

Passage (Q.106-Q.109):
There is a long-standing debate about the value of restoring natural environments.
Opponents say that we are not really able to return degraded landscapes to their
previous states. They claim that this perception risks more destruction because it
generates the expectation that things can always be put back together. This problem is
known as moral hazard. If restoration is feasible, then what’s to stop mining companies
from blowing mountains up and then just “repairing” them? On the opposite side of
the debate are pragmatists, who believe restoration efforts do more good than harm.
They’re not unconcerned about moral hazard, nor do they assert that humans are able
to recover landscapes to exactly as they once were. But, they say, if we can make
terrible situations better for people and nature alike, why not try?
This is the view taken by the Bronx River Alliance, a not-for-profit organisation that
has been engaged in restoring the Bronx River for the better part of a decade. After
centuries of being used as a dumping ground for industrial and residential waste, the
river can never be returned to its pre-colonial state, replete with thick forest along its
banks. Nor can we simply wish away the Kensico dam or the Cross- Bronx
Expressway.
But it is possible to make the Bronx River healthy. The Alliance has learned that the
key to doing this effectively is local involvement: to heal the river and keep it that
way, it must become meaningful in people’s lives. And the surest way for people to
feel that they have a stake in something is by acting onits behalf. From West Farms
and Hunts Point to Norwood and Williamsbridge, a network of Bronx volunteers
engages in outreach and education, monitors the river’s vitals and helps restock it with
fish.

106. Which of the following is most supported by the author’s argument?


(a) Restoration ecology is a useless practice as it will only promote more exploitative
behavior
(b) Restoration ecology is extremely important as it is the only way we can repair the
damage done to theEarth in a sustainable manner
(c) The Bronx river project is a failure as it can never be returned to its pre-colonial

54
Sensei Confidential

state
(d) One of the major reasons for the success of restoration ecology is the local
community involvement

107.Which of the following is an assumption the author makes?


a. If a broken thing can be mended then people may use it as an excuse to break other
similar things
b.Only natural surroundings that are destroyed by human involvement should be
restored back to their original self
c. Restoration ecology has a significant impact on the nature of the surrounding areas
of such a place
d.At least some members of a local community step in and take it upon
themselves to fix thedepreciation of natural resources

108.Which of the following can be inferred from the passage?


a. There exist many projects where restoration is impossible
b.The supporters of the movement don’t aim to recover the whole of the
natural scenario but just do some improvements
c. The Bronx river may never be fully brought back to its original setting
d.Unless the nature is properly promoted, people may not be very excited to
join in on restoring theenvironment

109.Which of the following, if true, would seriously undermine the author’s argument?
a. Majority of the nations across the world do not have such restoration projects
b.The opponents of this practice are many more than its supporters
c. The Bronx river was much cleaner before the local community decided to clean it
themselves
d.The Bronx river may have become cleaner had it been left undisturbed for 25
years

Passage (Q.110-Q.114): Disasters occur when people are affected by natural or technical
hazards – when lives are lost or property is destroyed. Research conducted in Sri Lanka
suggests that while heavy rainfall is the trigger for the flooding, the root causes of the

55
Sensei Confidential

disaster are social, in particular widespread poverty, conflict-induced migration and


problematic land-use practices. These characteristics are not homogeneous, meaning
different places and people are affected differently. The social characteristics of
communities are extremely important for hazard managers because they increase
peoples’ vulnerability to hazards.
Understandably, economically disadvantaged communities exposed to hazards have to
date received themost attention from Disaster Risk Reduction (DRR) specialists. This
is because hazards tend to harm predominantly those social groups that were already
disadvantaged before a disaster. Large focus has been placed on “underdeveloped” or
“developing” nations, where the economic disadvantage factors are particularly
obvious. However, examples, from affluent and less affluent countries, suggest the
need to consider economic vulnerability in more geographically and demographically
nuanced ways when implementing DRR activities. On the one hand, poorer
communities might bring alternative capabilities to DRR that are non-financial. On the
other hand, ignoring existing economic disadvantage within affluent contexts risks
significant loss of life and property, and forgoes the opportunity to improve the
circumstances of the affected sub-populations.
Across all stages of the disaster cycle – preparedness, response, and recovery –
knowledge about the nature and location of economically vulnerable groups is critical
for effective DRR. Before an event, knowing which groups have low levels of
preparedness is essential for planning tailored risk communication and support
initiatives. During a disaster, information on vulnerable groups can help to increase
the effectiveness of response measures, for example, by establishing priorities during
evacuations.

110. Which of the following, if true, weakens the authors claim?


(a) Sri Lanka is heavily prone towards natural disasters
(b) We do not have the appropriate technology to predict most natural disasters
(c) In majority of the affluent nations, the rich usually inhabit areas that are
more prone to destruction during disasters than any other areas
(d) Poorer nations are severely affected by natural disasters that take a toll on their
economy

111.All of the following can be inferred except?

56
Sensei Confidential

(e) DRR specialists need to create uniform plans that need to be enforced across all
nations irrespectiveof their affluence
(f) Previous surveys and information can help better prepare from natural disasters
(g) The trigger and root cause of natural disasters may be different
(h) The poorer regions have it worse when it comes to natural disasters

112. Which of the following statements will the author deem the truest?
(i) Mexico had an earthquake which was easily managed considering the
financial aid they received from international bodies.
(j) Cancun had various problems with floods as their government refused to rectify
the sewage systems
(k) Parts of Laos still haven’t recovered from the thunderstorm and majority of the
people in penury arestill homeless
(l) Kolkata has installed a new software that helps in calculating the impact of
natural disasters 24 hoursin advance

113.Which of the following conclusions can be properly drawn from the statements
above
a. The DRR specialists need to create disaster management dependent on the
financial situation of a country and identify different groups of people that
may be affected
b. Even developed nations have poor populations that may be similarly
affected as the people from developing nations
c. Both (A) and (B)
d. Neither (A) nor (B)

114.The author’s statement that “These characteristics are not homogeneous, meaning
different places andpeople are affected differently”-
a. Forms premise of the above passage
b. Forms conclusion of the above passage
c. Forms assumption of the author to the above passage
d. None of the above

Passage (Q.115-Q.119): A small boost in microlending to the developing world could lift

57
Sensei Confidential

more than 10.5 million people out of extreme poverty. Currently, 836 million people
– or 12% of the world’s population – experience extreme poverty, living off less than
US$1.25 a day. Using data from 106 developing countries from between 1998 and
2013 to examine the efficacy of microlending as a poverty-reduction tool, I found that
just a 10% increase in the gross microfinance loan portfolio per client could cut this
number by 1.26%. In 2017, the market for microfinance investments in micro, small
and medium enterprises, as well as the provision of financial services to those
businesses, is projected to grow by an average of 10% to 15%. Even stronger growth
is expected in India and the Asia-Pacific region.
Access to credit enables poor people to become entrepreneurs, increasing their earnings
and improving their quality of life. Many lenders accompany their small loans and
financial services with peer support, networking opportunities and even health care to
improve their clients’ odds of building a successful small business. In doing so, many
economists submit, they show that microfinance has a powerful potential to reduce
poverty. But evidence that microfinance actually works is mixed. Studies examining
its impact in rural Pakistan, urban Kenya and Uganda, among other developing
countries, have both confirmed and contradicted this premise: Microfinance is no
panacea. Numerous studies have shown that country-specific and cultural factors are
determinants in how microfinance will interact with poverty, and there are
occasionally devastating tales of failure in which the inability to repay a very small
loan has plunged households further into desperate penury. Overall, however, my
study suggests that more microcredit would benefit poor countries. National
governments and international development agencies can continue to promote
microfinance as a tool for reducing poverty, while bearing in mind the limitations of
any single strategy in tackling an entrenched global problem.

115.Which of the following, if true, would seriously undermine the author’s argument?
a. The poor population usually uses any sporadic money towards
satisfying their instant needs and desires
b. Governments that provided microfinances to their population only recovered
20% of what was lent
c. The private players responsible for micro financing received
ownership of many real estates asmanypeople were unable to repay
their debts.

58
Sensei Confidential

d. All of the above

116.Which of the following statements is in line with the author’s line of thoughts?
a. Microfinancing cannot work in developed nations
b. Microfinancing can only work in developed nations
c. Microfinancing can only work if it is given a uniform colour
d. Microfinancing can never work if it is given a uniform colour

117.On the basis of information provided above what might be the reason, if true,
for failure of some of themicrofinancing schemes?
a. The target group of some schemes are sometimes misplaced
b. The social factors inherent in the community are sometimes ignored
c. The information pertaining to income strata as well as general financial
behavior of a country is often misplaced
d. All of the above

118.The statement “Not all microfinancing schemes are effective” is:


a. Probably True
b. Definitely True
c. Probably False
d. Definitely False

119.Which of the following, if true, supports the author’s conclusion?


a. The market for microfinance investments is not an ever-growing enterprise
that has failed to support certain economies
b. Poor people have keen business acumen and they only lack enough
finances to get their shop up-and-running
c. Some developing nations have failed to introduce the microfinancing scheme
d. Poverty reduction is an important goal for developing countries today

120.The Indian music industry is singing. Singing the songs of the future. A future in
which it would be riding the wave of technology and, in fact, like elsewhere in the
world contributing to the GDP. The futureis bright because it is not just physical

59
Sensei Confidential

sales that had been threatened by piracy. The digital revolution is bringing all the
players onto one platform. .

The above question has a paragraph from which the last sentence is deleted.
From the options given, choose the one that completes the paragraph in the most
appropriate way.
a. Technology is something that this industry has been forced into.
b. This augurs well for an industry that has been in a slump for some time now.
c. The slump in the industry is also reflected in the sale of hardware.
d. To understand this, all you need to do is walk round the marketplace;
nobody buys music systems or speakers

121.The Indian postal service is badly maintained. Twenty years ago, first-class letter
delivery cost only threerupees. Since then, the price has increased severely, with
an actual decrease in the reliability of service. Which of the following weakens the
above argument?
a. The volume of mail handled by the postal service has increased dramatically over
the last thirty years.
b. Unprecedented increases in the cost of fuel for trucks and planes have put
severe upward pressureson postal delivery costs.
c. The average level of consumer prices overall has decreased fourfold over the last
thirty years.
d. Private delivery services usually charge more for comparable delivery services

122.In a certain code, BREAKTHROUGH is written as EAOUHRBRGHKT. How is


DISTRIBUTION written inthat code?
a. TISTBUONDIRI
b. STTIBUONRIDI
c. STTIBUDIONRI
d. RISTTIBUDION

123.If the first and the second letters of the word UNPRECEDENTED are
interchanged with the last and the second last letters and similarly, the third and

60
Sensei Confidential

the fourth letter are interchanged with the third and the fourth letter from the last
respectively, then what will be the seventh letter to the right of the third letter
from the left?
a. C
b. E
c. P
d. R

124.Pointing to a lady, Vikram said, "she is the only daughter of my father-in-


law's wife". How is this lady related to Vikram?
a. Mother
b. Wife
c. Cousin
d. Sister

Passage (Q.125 - Q.129): Deforestation has historically been the price of


development, but the world is now going through a forest transition; since 2015, there
has been net global reforestation. The pace and quality of this transition is mixed. In
the world’s remaining high conservation-value forests, deforestation rates are high and
poverty persists but development opportunities are within sight. These forests are
mainly located in the tropical developing world and have growing human populations.
Because as forest- dependent people are increasingly involved in cash economies, they
use their forests to participate in markets. This will inevitably lead to changes in
forests.
Conservationists typically respond to this in one of two ways. They either tackle the
threat head on andtry to counter it or they hand over forest management to local
people. But neither threat-based conservation nor local management has proven
successful for preserving forests. Tropical forests are still subject to high deforestation
rates in less developed countries and conservationists lament the ongoing decline in
biodiversity and ecosystems.
To local people, tackling threats to forests is seen as opposing development and ergo it
will continue to fail. Opposing new roads in areas where people are deprived of

61
Sensei Confidential

development opportunities, for instance, is clearly not a viable way forward. But the
alternative strategy of handing management over to local people in the hope that they
will protect biodiversity is also not pragmatic. If the only options on offer are forest
protection or development at their expense (plantations, for instance, or mines and
agriculture) then most people will, naturally, choose the latter.
It’s mindful that local peoples’ needs and aspirations change with time. People living
near forests will say their preferred future includes the continued existence of forests
and therefore, biodiversity and intact ecosystems. The challenge is to achieve this
alongside improvements in livelihoods.

125.On the basis of information provided above, why would the local people fail in
preserving forests?
a. Because the inherent nature of people is self-serving and everyone wants to
live comfortably
b. Because for the local people, intervention is bad and ought to be stopped
c. Because the local people hate to share their resources with
others which results in de-commercialization of forests
d. Both (A) & (B)

126.Which of the following statements is in line with the author’s line of thoughts?
a. Deforestation and reforestation is happening all around but the movement is
sporadic and in need ofan analysis
b. Human population and forests can be separated but the local population need
more convincing
c. Developing countries have more deforested land than developed countries
d. Local population can make forest conversation if they are paid monthly wages
for the same

127.Which of the following, if true, would seriously undermine the author’s argument?
a. Many conservationists are actually pro-local people and believe in leaving the
forests in their hands
b. The government has enough resources to form a group of forest
rangers that may protect theaffected areas

62
Sensei Confidential

c. Big corporations are the major threat to deforestation in less developed


countries
d. None of the above

128.The author’s statement that “To local people, tackling threats to forests is seen as
opposing development and ergo it will continue to fail”-
a. Forms premise of the above passage
b. Forms conclusion of the above passage
c. Forms assumption of the author to the above passage
d. None of the above

129.What could be an apt heading for the passage?


a. Forests are the lungs of the Earth
b. Once green is today’s obscene
c. Conserving forests: beyond ideas of local integration
d. Local communities are their life

Directions (Q.130 - Q.134): Study the following information carefully and answer
the given questions: A, B, C, D, E, F, G and H are sitting around a square able in
such a way that four of them sit at four corners of the square while four sit in the
middle of each of the four sides. The ones who sit at the four corners face the centre
while those who sit In the middle of the sides face outside.
Two females sit in the middle of the sides and two at the corners, A sits second to the left
of G. G sits Inthe middle of one of the sides. C sits fourth to the right of his wife and his
wife is not an immediate neighbour of A or G. B sits third to right of her husband. B does
not sit at any of the corners. Only D sits between B and H. H is the husband of A. E is a
male.

130. Which of the following is true with respect to the given seating arrangement?
a. No two males are immediate neighbours of each other
b. G and H do not face each other in the seating arrangement
c. E and D are immediate neigh-hours of each other
d. A sits in the centre of one of the sides of the square table

63
Sensei Confidential

131. Who amongst the following is B's husband?


a. C (b) G (c) E (d) F

132. How many people sit between B and C when counted in anti-clockwise direction
from B?
a. None (b) One (c) Two (d) Three

133. Who amongst the following is the wife of C?


a. D (b) F (c) B (d) G

134. What is the position of E with respect to C?


a. Immediately to the left
b. Second to the left
c. Third to the right
d. Second to the right

Directions(Q.135 and Q.136): Read the following passage and answer the items that follow :

A, B, C, D, E and F are cousins. No two cousins-are of the same age, but all have birthdays
on the same day of the same month. The youngest is 17 years old and the oldest E is 22 years
old. F is somewhere between E and D in age. A is older than B, C is older than D, A is one
year older than C

135. Which one of the following is possible?


(a) D is 20 years old
(b) F is 18 years old
(c) F is 19 years old
(d) F is 20 years old

136. What is the number of logically possible orders of all six cousins in
terms of increasing age?
(a) 1

64
Sensei Confidential

(b) 2
(c) 3
(d) 4

65
Sensei Confidential

QUANTITATIVE APTITUDE

Direction(137- 141). The following table gives the sales of batteries manufactured by company
over the years. Study the table and answer the question that follows:
NUMBER OF DIFFERENT TYPES OF BATTERIES SOLD BY A COMPANY OVER THE
YEARS (NUMBERS IN THOUSAND)

Q.137. The total sales of all the seven years are the maximum for which battery?
(A) 4AH
(B) 7AH
(C) 32AH
(D) 35AH

Q.138. What is the difference in the number of 35AH batteries sold in 1993 and 1997?
(A) 24000
(B) 28000
(C) 35000
(D) 39000

Q.139. The percentage of 4AH batteries sold to the total number of batteries sold was maximum
in the year :
(A) 1994
(B) 1995
(C) 1996

66
Sensei Confidential

(D) 1998
Q.140. In the case of which battery there was a continuous decrease in sales from 1992 to 1997?
(A) 4AH
(B) 7AH
(C) 32AH
(D) 35AH

Q.141. What was the approximate percentage increase in the sales of 55AH batteries in 1998
compared to that in 1992 ?
(A) 28%
(B) 31%
(C) 33%
(D) 34%

Direction(142-146): The bar graph given shows the foreign exchange reserves of a country (in
million US $) form 1991-92 to 1998-99. Answer the question based on this graph.

Q.142. The foreign exchange reserves in 1997-98 was how many times that in 1994-95?
(A) 0.7
(B) 1.2
(C) 1.4

67
Sensei Confidential

(D) 1.5

Q.143. What was the percentage increase of foreign exchange reserves over the previous year,
is the highest?
(A) 100
(B) 150
(C) 200
(D) 620

Q.144. For which year, the percentage increase of foreign exchange reserves over the previous
year, is the highest?
(A) 1992-93
(B) 1993-94
(C) 1994-95
(D) 1996-97

Q.145. The foreign exchange reserves in 1996-97 were approximately what percent of the
average foreign exchange reserves over the period under review?
(A) 95%
(B) 110%
(C) 115%
(D) 125%

Q.146. The ratio of the number of years, in which the foreign exchange reserves are above the
average reserves, to those in which the reserves are below the average reserves, is:
(A) 2 : 6
(B) 3 : 4
(C) 3 : 5
(D) 4 : 4

Directions for Questions 147 148: These Questions are based on the following table.

POPULATION OF FOUR METRO CITIES AT THE BEGINNING OF THE YEAR 1990

68
Sensei Confidential

147. If Bombay were to become the most populous city at the end of the year, what is the least
number of people who must shift to Bombay during the year, given that, every year 1% of its
population shifts to other cities (Assume that the population of other metro does not change)?
a. 6
b. 1,04,64,301
c. 1,05,706
d. 1,05,705

148. If the most populous city has 40% female population, and the least has 35% female
population, then the difference between male population of these two cities is closest to
a. 60 lakh
b. 57 lakh
c. 17.9 lakh
d. 56 lakh

Directions for questions149 and 150: These questions are based on the graph given below,
which shows the number of deaths due to fire accidents in the summer of 1992.

69
Sensei Confidential

149. The average number of deaths per day during the given period is
a. 22.04
b. 22.5
c. 23.09
d. 21.08

150. Out of the total deaths from 30 th March to 8th May, if 22% occurred in place X and 10%
of them are because of cigarettes, then the number of fire accidents for the given period in place
X because of cigarettes is approximately.
a. 19
b. 20
c. 22
d. 23

70

You might also like